Week 8

Réussis tes devoirs et examens dès maintenant avec Quizwiz!

For each numbered items select the one lettered option which is most closely associated with it (each lettered option can be selected once, more than once, or not at all). A) Tubocurarine B) Cisatracurium C) Mivacurium D) Succinylcholine E) Dantrolene F) Gabapentin G) Diazepam H) Tizanidine I) Baclofen 1) This drug can block Nn receptors in the autonomic ganglia 2) These drugs are quickly metabolized by plasma cholinesterase. 3) This drug is an alpha-2 agonist 4) This drug is a GABAB receptor agonist 4) This drug is a GABAA receptor agonist

1. tubocurarine. Tubocurarine is the only listed drug that can also block Nn receptors and therefore can act as a ganglionic blocking drug, mainly when given in larger doses. Others neuromuscular blockers like cisatracurium and mivacurium are devoid of ganglionic blocking activity and succinylcholine can activate Nn receptors, when given in larger doses. 2. Succinylcholine and mivacurium are rapidly metabolized by plasma cholinesterase and therefore their duration of action after a standard therapeutic dose is brief (succinylcholine .8 min, mivacurium . 15 min). 3. Tizanidine is an alpha-2 agonist that activates both presynaptic and postsynaptic receptors in the CNS. This reduces the release of excitatory neurotransmitters (mainly glutamate) in the spinal cord, which in turn decreases the excitatory input to motor neurons. 4. Baclofen is a GABAB receptor agonist. Activation of GABAB receptor in the CNS causes hyperpolarization likely by increasing K+ conductance. This in turn leads to a decreased release of excitatory transmitters both in the brain and in the spinal cord.

10 An epidemiologic study of postmenopausal women is performed. The subjects undergo periodic examination by dual-energy x-ray absorptiometry (DEXA) scan performed on the hip and lumbar vertebrae to evaluate bone mineral density over the next 10 years. They respond to a survey regarding their past and present use of drugs, diet, activity levels, history of bone fractures, and medical conditions. A cohort of the subjects is identified whose bone mineral density is closest to that of the young adult reference range and in whom no bone fractures have occurred. Which of the following strategies is most likely to be supported by the study data to provide the best overall long-term reduction in risk of fracture in postmenopausal women? A Increasing bone mass with exercise during young adulthood B Limited alcohol use, and avoidance of the use of tobacco C Initiation of estrogen replacement therapy after a fracture D Supplementation of the diet with calcium and vitamin D after menopause E Use corticosteroid therapy for inflammatory conditions

10 An epidemiologic study of postmenopausal women is performed. The subjects undergo periodic examination by dual-energy x-ray absorptiometry (DEXA) scan performed on the hip and lumbar vertebrae to evaluate bone mineral density over the next 10 years. They respond to a survey regarding their past and present use of drugs, diet, activity levels, history of bone fractures, and medical conditions. A cohort of the subjects is identified whose bone mineral density is closest to that of the young adult reference range and in whom no bone fractures have occurred. Which of the following strategies is most likely to be supported by the study data to provide the best overall long-term reduction in risk of fracture in postmenopausal women? A Increasing bone mass with exercise during young adulthood B Limited alcohol use, and avoidance of the use of tobacco C Initiation of estrogen replacement therapy after a fracture D Supplementation of the diet with calcium and vitamin D after menopause E Use corticosteroid therapy for inflammatory conditions A The total bone mass is an important determinant of the subsequent risk of osteoporosis and its complications. A proactive regimen of exercise that puts stress on bones to increase mass before the inevitable loss after age 30 years is most likely to reduce the subsequent risk of osteoporosis. Alcohol and tobacco use are not major risks for osteoporosis. Postmenopausal estrogen or raloxifene therapy can help preserve bone mass; however, by the time a fracture has occurred, there has already been significant bone loss. Likewise, dietary supplements after menopause are not harmful, but at best only partially slow the loss of bone that accompanies aging. Corticosteroid therapy is just one of many risk factors for osteoporosis, but short courses of corticosteroids have minimal effects on bone formation.

10) Which of the following drug classes can enhance the neuromuscular blockade produced by nondepolarizing neuromuscular blocking drugs? A) Halogenated inhalational anesthetics B) Cholinesterase inhibitors C) Antimuscarinic drugs D) Cholinergic agonists E) Alpha-1 agonists

10) Answer: A Many halogenated inhalational anesthetics decrease the sensitivity of the postjunctional membrane to depolarization, and therefore act synergistically with the nondepolarizing neuromuscular blocking drugs. B) Cholinesterase inhibitors increase the availability of acetylcholine and therefore would antagonize, not enhance, the action of nondepolarizing neuromuscular blocking drugs that are competitive antagonist of acetylcholine. C, D, E) These drugs do not act on the skeletal muscle and therefore cannot interact with nondepolarizing neuromuscular blocking drugs.

12 An 82-year-old man has had progressively worsening lower back, bilateral hip, and right shoulder pain for the past 6 years. He reports that he has had to buy larger hats. On physical examination, there is no joint swelling, erythema, warmth, or tenderness, but the range of motion is reduced. Radiographs show narrowing of joint spaces with adjacent bony sclerosis. A skull radiograph shows thickening but diminished density of the skull bones. A bone biopsy specimen at the iliac crest shows a loss of normal trabeculae, with a mosaic pattern and increased numbers of osteoclasts and osteoblasts. Which of the following complications is the patient most likely to experience as a result of this condition? A Ankylosing spondylitis B Enchondromatosis C Fibrous dysplasia D Osteoid osteoma E Osteosarcoma

12 E In 5% to 10% of patients with severe polyostotic Paget disease, sarcomas (including osteosarcoma) can arise years later in bone affected by the disease. Mutations in SQSTM1 are found in both familial and sporadic cases of Paget disease of bone. Ankylosing spondylitis involves the spine and carries no risk of malignancy. An enchondroma does not arise in the setting of Paget disease. Fibrous dysplasia is a focal developmental defect of bone seen at a younger age. An osteoid osteoma is a small benign cortical bone lesion that can produce severe pain in children and young adults.

12) A 75-year-old man must undergo abdominal surgery to remove a colonic carcinoma. The patient has severely impaired hepatic and renal function and is known to have a genetic deficiency in plasma pseudocholinesterase activity. Which of the following would be most likely the best skeletal muscle relaxant agent for this patient? A) Succinylcholine B) Dantrolene C) Tubocurarine D) Cisatracurium E) Mivacurium

12) Answer: D (Katzung, pp 432, Brunton, pp 228) Cisatracurium is a neuromuscular blocking drug that has the unique property of being inactivated primarily by a form of spontaneous breakdown (also known as Hoffman elimination). Because of this, it does not exhibit an increase in half life in patients with compromise hepatic or renal function and it is therefore the agent of choice under these conditions. A, E) The patient has a genetic deficiency in plasma pseudocholinesterase activity. Since succinylcholine and mivacurium are inactivated by plasma cholinesterase, these drugs would have an unacceptable long duration of action in this patient. B) Dantrolene cannot cause skeletal muscle paralysis (unless huge doses are given ) and therefore is not used as a skeletal muscle relaxant during surgery. C) Tubocurarine is metabolized by the liver (60%) and eliminated by the kidney (40%). Therefore it would have a very long half life in this patient.

13) A 55-year-old woman is undergoing major surgery to remove a breast carcinoma. The woman, who is an heavy smoker, has been suffering from chronic obstructive pulmonary disease for 15 years. A drug is administered preoperatively and the anesthesia is supplemented with a skeletal muscle relaxant. Which of the following drugs would be contraindicated for this patient? A) Tubocurarine B) Glycopyrrolate C) Clonidine D) Diazepam E) Cisatracurium

13) Answer: A (Katzung, pp 438, Brunton, pp 226) Tubocurarine can cause histamine release from mast cells. A prominent effect of histamine is bronchoconstriction and therefore tubocurarine should not be used in a patient with asthma or COPD. B,)Glycopyrrolate is often given as a preoperative medication in order to prevent effects related to vagal activation (sialorrhea and bradycardia) C, D) Clonidine and diazepam are sometimes given as a preoperative medication because both have sedative properties and potentiate the action of anesthetic agents. E) This nondepolarizing neuromuscular blocking drug could be used since it does not release histamine.

16) A 45-year-old woman recently diagnosed with spasmodic torticollis received multiple injections of a drug into the dystonic muscles of the neck. The therapy improved the position of the head and reduced painful muscle spasms for 3 months after the injections. Which of the following drugs did the patient most likely receive? A) Succinylcholine B) Mivacurium C) Tubocurarine D) Botulin toxin E) Dantrolene

16) Answer: D (Katzung, pp 444, Brunton, pp 229) Spasmodic torticollis is an involuntary tonic contraction of the neck muscles causing painful spasm and abnormal head position. In severe cases the selective denervation of the neck muscle is the most successful surgical approach but injection of botulin toxin into the dystonic muscles of the neck can reduce the symptoms for 1 to 3 months in about 70% of patients . Botulin toxin inhibits acetylcholine release from cholinergic terminals, so weakening skelemuscle contractions. A, B, C) Neuromuscular blocking drugs are of no value in this disease mainly because of their short duration of action. E) Spasmolytic drugs (dantrolene, benzodiazepines, tizanidine, etc.) are sometimes used but they decrease spasms in only about 25% of cases.

20) A 65-year-old woman underwent hysterectomy to remove a uterine carcinoma. The anesthesiologist chose thiopental sodium for induction and isoflurane and tubocurarine for maintenance of general anesthesia. The anesthesiologist also administered another drug in order to counteract tubocurarine induced hypotension. A drug belonging to which of the following classes was most likely given? A) Beta-1 agonist B) D1 antagonist C) Muscarinic agonist D) Cholinesterase inhibitor E) H1 antagonist

20) Answer: E (Katzung, pp 438, Brunton, pp 226) Tubocurarine induced hypotension is mainly due to histamine release and premedication with an antihistamine drug is often used in order to attenuate this adverse effect. A, B, C, D) Drugs from these classes would increase, not decrease, tubocurarine-induced hypotension.

42 A 55-year-old, previously healthy man has had episodes of pain and swelling of the right first metatarsophalangeal joint for the past year. These flare-ups usually occur after consumption of alcohol, typically port wine (Six Grapes). On physical examination, there is exquisite tenderness with swelling and erythema of the right first metatarsophalangeal joint. A joint aspiration is performed, and polarized light microscopy (arrow in axis of red compensator) of the fluid obtained shows the finding in the figure, and many neutrophils in a small amount of fluid. Which of the following drugs is most likely to be useful as the first line of treatment? A Antibiotic B Corticosteroid C Nonsteroidal anti-inflammatory agent D Folate antagonist E Xanthine oxidase inhibitor 413 image

42 C Acute inflammation of the first MP joint, caused by precipitation of the needle-shaped negatively birefringent (yellow) uric acid crystals in the joint space, is typical of gout. Hyperuricemia is a sine qua non for the development of gout. Not all patients with hyperuricemia develop gout, and not all patients with gout have hyperuricemia, however. Other, ill-defined factors play a role in pathogenesis. Involvement of the big toe is classic, but other joints may be involved. Although attacks of gout are often precipitated by a heavy bout of alcohol consumption, liver damage (marked by elevated transaminases) is not a feature of gouty arthritis. Nonsteroidal anti-inflammatory drugs (NSAIDs) represent the first line of therapy for acute attacks. This is a chemically induced inflammatory reaction, not infectious, so antibiotics are not indicated. Glucocorticoids have a more pronounced effect upon chronic inflammatory conditions, and their continued use in joints will lead to degenerative arthritic changes. Methotrexate may be considered for treatment of rheumatoid arthritis. Xanthine oxidase inhibitors reduce uric acid formation and uricosuric agents help increase uric acid secretion, but they do not work acutely.

6) Which of the following statements correctly pairs the skeletal muscle relaxant with the drug that can antagonize its muscle relaxant effect? A) Cisatracurium - flumazenil B) Succinylcholine - atropine C) Tubocurarine - neostigmine D) Dantrolene - tizanidine E) Botulinum toxin - baclofen

6) Answer: C (Katzung, pp 440, Brunton, pp 227) All nondepolarizing neuromuscular blockers (tubocurarine, cisatracurium, etc.) cause a competitive blockade of Nm receptors. Their action is antagonized by acetylcholine as well as by drugs that increase the availability of acetylcholine at the motor end plate (cholinesterase inhibitors, like neostigmine). This antagonism is exploited clinically and neostigmine is often used to speed up the recovery from the neuromuscular blockade remaining after completion of surgery. A) Flumazenil can antagonize the muscle relaxation induced by benzodiazepines, not by Cisatracurium. B) Atropine has no blocking effect on Nm receptor. D, E) All the listed drugs cause muscle relaxation, so they can enhance, not antagonize, each other.

8) A direct consequence of the mechanism of action of tubocurarine is the blockade of which of the following membrane potentials? A) Action potential of motor nerve B) Miniature end plate potential C) Resting potential of smooth muscle D) Action potential of cardiac muscle E) Action potential of corticospinal tract

8) Answer: B (Brunton, pp 223, Golan pp 94) The formation of miniature end plate potential is caused by the release of small quanta of acetylcholine that activate Nm receptors at the motor end plate. By blocking these receptors tubocurarine block the formation of this potential. A, D, E) Neuromuscular blocking drugs have negligible effects on the formation of action potentials in the motor neuron, cardiac muscle or corticospinal tract. C) Smooth muscle membrane has no Nm receptors. Therefore neuromuscular blocking drugs have no direct effect on smooth muscle.

39 A 48-year-old woman had chronic pain of the left shoulder and right hip for 8 months. The pain resolved within 1 month. Two months later, she developed pain in the right knee and ankle, which resolved within 6 weeks. On physical examination, she is now afebrile. There is pain on movement of the left shoulder and right hip. A radiograph of the left arm shows extensive bony erosion of the humeral head. A biopsy specimen of synovium is taken and on microscopic examination shows a marked lymphoplasmacytic infiltrate and arteritis with endothelial proliferation. Which of the following infectious agents is most likely responsible for these findings? A Borrelia burgdorferi B Group B streptococcus C Mycobacterium tuberculosis D Neisseria gonorrhoeae E Treponema pallidum

9 A Her chronic arthritis is the late, or stage 3, manifestation of Lyme disease, which, as in this case, tends to be remitting and migratory, involving primarily the large joints. The presence of a lymphoplasmacytic infiltrate with endothelial proliferation is characteristic (but not diagnostic) of Lyme arthritis. The infectious agent, Borrelia burgdorferi, is a spirochete that is spread by the deer tick (Ixodes). This stage is reached about 2 to 3 years after the initial tick bite, and joint involvement can appear in about 80% of patients. Group B streptococcus may produce an acute osteomyelitis or arthritis in neonates. Tuberculous arthritis may involve large, weight-bearing joints, and it can be progressive, leading to ankylosis. The histologic features of tuberculous arthritis include caseating granulomas. Neisseria gonorrhoeae can cause an acute suppurative arthritis. In both conditions, the inflammatory infiltrate contains a preponderance of neutrophils. Treponema pallidum may produce gummatous necrosis that can involve large joints, and there can be lymphoplasmacytic infiltrates with endarteritis, but syphilitic arthritis is quite rare.

In osteogenesis imperfecta, the primary defect appears to lie with the A) osteoblasts B) osteoclasts C) osteocytes D) chondrocytes E) chondroblasts

A

46 A 33-year-old woman has been bothered by a bump on the dorsum of her left wrist for the past 4 months. On physical examination, there is a 1-cm firm but fluctuant subcutaneous nodule over an extensor tendon of the left wrist. The nodule is painful on palpation and movement. Mucoid fluid is aspirated from the nodule. What is the most likely diagnosis? A Ganglion cyst B Giant cell tumor C Lipoma D Nodular fasciitis E Rheumatoid nodule F Tophus 414

A A ganglion cyst has a thin wall and clear, mucoid content. It arises in the connective tissue of a joint capsule or tendon sheath. The extensor surfaces of the hands and feet are the most common sites, particularly the wrist. Ganglion cysts probably arise after trauma from focal myxoid degeneration of connective tissue to produce a cystic space. They may regress. If not, and if they are painful, they can be excised. Tenosynovial giant cell tumor (villonodular synovitis) is a more diffuse form of giant cell tumor of tendon sheath (a solid mass lesion) and is a proliferation of mononuclear cells resembling synoviocytes. A lipoma is a mass of adipocytes and is not cystic. Nodular fasciitis is a solid reactive fibroblastic proliferation seen in the upper extremities and trunk of young adults, sometimes occurring after trauma. Rheumatoid nodules are firm, solid masses that typically occur in individuals who already have joint involvement with rheumatoid arthritis. A tophus is a solid mass of chalky sodium urate crystals in patients who have a history of gout.

3 A 29-year-old woman, G3, P2, gives birth to an infant following an uncomplicated pregnancy. The infant's height is below the fifth percentile. On physical examination, the infant's torso and head size are normal, but the extremities are short. The forehead appears prominent. Radiographs show short, slightly bowed long bones, but no osteopenia. The other two children in the family are of normal height. The affected child has no difficulty with activities of daily living after modifications are made in the home and school for short stature, and later becomes a physician. Which of the following conditions is likely to be present in this child? A Achondroplasia B Hurler syndrome C Osteogenesis imperfecta D Rickets E Scurvy F Thanatophoric dysplasia

A Achondroplasia is most often the result of a spontaneous new mutation in the fibroblast growth factor receptor 3 (FGFR3) gene, leading to abnormal cartilage proliferation at growth plates and affecting mainly endochondral bone growth. The homozygous form is lethal in utero. In Hurler syndrome, mucopolysaccharidosis type I, children are normal at birth but then develop growth retardation, mental retardation, hepatosplenomegaly, and joint stiffness. Osteogenesis imperfecta may manifest at birth with multiple fractures from severe osteopenia as a result of abnormal type I collagen synthesis. Rickets may occur in childhood from vitamin D deficiency, producing deficient bone mineralization, but dwarfism is not a feature. Vitamin C deficiency causes scurvy and can lead to abnormal bone matrix with mild deformity, but not dwarfism. Thanatophoric dysplasia is the most common form of lethal dwarfism and also results from a mutation in FGFR3.

15 A 26-year-old woman has had malaise, arthralgias, and myalgias for the past 2 months. On physical examination, there is no joint swelling or deformity. Laboratory studies show a serum creatinine level of 3.9 mg/dL. A renal biopsy specimen shows a proliferative glomerulonephritis. She receives glucocorticoid therapy for 3 months. She now has left hip pain with movement. On physical examination, there is no swelling or deformity. A radiograph of the left leg and pelvis shows patchy radiolucency and density of the femoral head with flattening of the bone. A total replacement of the left hip is performed, and gross examination of the sectioned femoral head shows collapse of articular cartilage over a pale, wedge-shaped, subchondral area. What is the most likely diagnosis? A Avascular necrosis B Enchondroma C Osteoarthritis D Osteomyelitis E Renal osteodystrophy

A Avascular necrosis of bone (osteonecrosis) represents a localized area of bone infarction, most often in a metaphyseal medullary cavity or subchondral epiphyseal location. The femoral head is most often affected. Underlying conditions associated with osteonecrosis include hemoglobinopathies (sickle cell disease in particular), fracture, barotrauma, hypercoagulable states, and hyperlipidemia. Glucocorticoid therapy decreases osteoblastogenesis to promote avascular necrosis, as in this patient with systemic lupus erythematosus and glomerulonephritis. An enchondroma is a benign tumor of hyaline cartilage that arises in the medullary space of young adults. Osteoarthritis may produce some cartilaginous erosion, but not collapse or bone infarction. Osteomyelitis typically is not so localized, and there is irregular new bone formation (involucrum). The patient's course is quite short for renal osteodystrophy, which is mediated through chronic renal failure and produces lesions such as osteitis fibrosa cystica.

9 A 35-year-old woman with active lupus nephritis falls forward and lands on her left hand. She has immediate pain. On examination there is crepitus at the wrist. A radiograph shows radial and navicular fractures along with marked osteopenia. Which of the following medications most likely contributed to the fracture? A Hydrocortisone B Ibuprofen C Lisinopril D Losartan E Methotrexate

A Corticosteroids may be needed to treat severe lupus nephritis, and continuing therapy increases the risk for Cushing syndrome with osteoporosis. Corticosteroids stimulate RANKL expression, inhibit osteoblast osteoprotegerin (OPG) synthesis, and thereby enhance osteoclast proliferation to promote bone resorption. Ibuprofen and other nonsteroidal anti-inflammatory drugs (NSAIDs) should not be used with renal failure. Lisinopril, an angiotensin-converting enzyme (ACE) inhibitor, and losartan, an angiotensin receptor blocker (ARB), may be used to treat hypertension with renal disease, but they do not directly affect bone density. Methotrexate is a chemotherapy agent that is best used with rheumatoid arthritis, not systemic lupus erythematosus, and at low doses is less likely to lead to osteoporosis.

34 A 10-year-old girl has developed worsening pain in the knees and ankles for the past 3 months and now has difficulty walking. On physical examination, these joints are swollen and warm to the touch, and there is diffuse muscle tenderness. She has a temperature of 39.2° C. There is an erythematous skin rash across the bridge of her nose and on the dorsa of her hands. Hepatosplenomegaly is present, and a friction rub is audible on auscultation of her chest. A joint aspirate is obtained from the left knee, and a microbiological culture of the fluid is negative. On microscopy, the joint fluid has increased numbers of lymphocytes, but few neutrophils. Her condition improves over the next year, and she has no residual joint deformity. Which of the following laboratory findings is most likely to be found in this patient? A ANA serologic titer of 1:1024 B Borrelia burgdorferi serology positive C Chlamydia trachomatis urine culture positive D Ferritin level in serum of 7245 ng/mL E Hemoglobin S on hemoglobin electrophoresis F Rheumatoid factor serologic titer of 1:512 G Serologic test for syphilis (STS) positive H Uric acid level in serum of 15.8 mg/dL

A Juvenile rheumatoid arthritis (RA), or juvenile idiopathic arthritis (JIA), in contrast to the adult type of rheumatoid arthritis, is more likely to be self-limited and nondeforming. JIA typically is rheumatoid factor negative, but ANA positive. JIA is more likely than the adult form to have systemic manifestations, such as rash, myalgia, myocarditis, pericarditis, uveitis, and glomerulonephritis. A positive serologic test for Borrelia burgdorferi is seen in Lyme disease, which tends to be associated with migratory arthritis of large joints. Similar to JIA, Lyme disease produces a chronic deforming arthritis in only about 10% of cases. Chlamydia trachomatis is typically the agent that produces the nongonococcal urethritis seen with reactive arthritis, which, similar to other spondyloarthropathies, most commonly involves the sacroiliac joint. Ferritin levels are markedly increased in hereditary hemochromatosis, in which iron deposition in joints can produce a chronic arthritis similar to osteoarthritis or pseudogout. Sickle cell disease with hemoglobin S can lead to aseptic necrosis, often of the femoral head, and to bone infarcts, with chronic arthritis secondary to bone deformity. Rheumatoid arthritis tends to be recurrent and causes progressive joint deformities, typically of hands and feet. Congenital syphilis can produce periosteitis and osteochondritis with bone deformities; tertiary syphilis in adults can produce gummatous necrosis with joint destruction or loss of sensation, particularly in the lower extremities, leading to repeated trauma that deforms joints (Charcot joint). Some cases of gouty arthritis are accompanied by hyperuricemia; gouty arthritis tends to manifest as an acute attack in older individuals.

25 A 45-year-old man has experienced pain in the area of the left hip and upper thigh for the past 7 months. On physical examination, there is tenderness on deep palpation of the left side of the groin. The range of motion at the left hip is decreased, but there is no swelling or warmth on palpation. Pelvic and left leg radiographs show an upper femoral metaphyseal mass lesion that erodes into the surrounding bone cortex. The proximal femur is excised and on sectioning has the gross appearance shown in the figure. Which of the following cell types is most likely to be proliferating in this mass? A Chondrocyte B Osteoblast C Osteoclast D Plasma cell E Primitive neuroectodermal cell

A The glistening, gray-blue appearance shown in the figure is typical of cartilage, and this lesion most likely represents neoplastic proliferation of chondrocytes. This chondrosarcoma has infiltrated the medullary cavity and invaded the overlying cortex, characteristics of a malignant process. Most chondrosarcomas are low grade. They occur in a broad age range, in contrast to many other primary bone tumors that tend to occur in the first two decades of life. Most chondrosarcomas arise toward central portions of the skeleton. Osteoblast proliferation may be seen in a small tumor mass of the cortex known as osteoid osteoma, whereas a larger mass in the axial skeleton may be called an osteoblastoma. Osteosarcomas are derived from osteoblasts, but are malignant. They are usually seen in younger individuals and do not have a bluish-white appearance because they are marked by osteoid production. Giant cell tumors arise during the third to fifth decades; they involve epiphyses and metaphyses. Grossly, they are large, red-brown, cystic tumors. Giant cells resembling osteoclasts are present in giant cell tumors of bone. These tumors are believed to arise from cells of monocyte-macrophage lineage. Atypical plasma cells appear with multiple myeloma. Myelomas are dark red, rounded, lytic lesions that are often multiple. Primitive neuroectodermal cells are present in a Ewing sarcoma.

43 A 51-year-old man has endured episodes of intense local pain involving his left foot for the past 4 months. The pain may last hours to days. Physical examination identifies the right metatarsophalangeal (MP) joint as the focus of tenderness and swelling, but minimal loss of joint mobility. A painless 2-cm nodule with overlying ulcerated skin is present on the lateral aspect of the MP joint. Beneath the eroded skin is a chalky white deposit of soft material. A firm 1-cm subcutaneous nodule on the extensor surface of the left elbow is excised and has the microscopic appearance shown in the figure. Which of the following mechanisms is most important in causing joint injury in this man? A Activation of neutrophils by phagocytosis of urate crystals B Release of TNF causing acute joint inflammation C Deposition of serum cholesterol into the synovium D Granulomatous inflammation with Mycobacterium tuberculosis infection E Reduced metabolism of homogentisic acid

A The histologic picture is that of a central amorphous aggregate of urate crystals surrounded by reactive fibroblasts and mononuclear inflammatory cells. This is a gouty tophus. Tophi are large collections of monosodium urate crystals that can appear in joints or soft tissues of patients with gout. Large superficial tophi can erode the overlying skin. Precipitation of urate crystals into the joints produces an acute inflammatory reaction in which neutrophils and monocytes can be found. Neutrophils phagocytize urate crystals, which cannot be digested, but cause release of destructive neutrophilic lysosomal enzymes and oxygen free radicals. Release of crystals from the neutrophils perpetuates this cycle of inflammatory response. Inflammation of the joints involves different mechanisms depending on the etiology. In rheumatoid arthritis, release of tumor necrosis factor (TNF) by macrophages plays a central role, as evidenced by the dramatic relief provided by anti-TNF agents. Marked hypercholesterolemia, as occurs in familial hypercholesterolemia, can lead to deposition of cholesterol in tendons and elsewhere. When deposited in tendons, the yellowish lesions are called xanthomas; the cholesterol crystals appear microscopically as clefts in the tissue. Extrapulmonary Mycobacterium tuberculosis infection can cause granulomatous inflammation and chronic arthritis and skin lesions, but there is caseous necrosis with epithelioid cells and no urate crystals. Tuberculous arthritis, in contrast to gouty arthritis, almost never begins in the MP joint. Reduced metabolic breakdown of homogentisic acid occurs in the inborn error of metabolism known as alkaptonuria, and deposition of homogentisic acid (ochronosis) in cartilage causes an arthritis that typically affects large joints

49 A 36-year-old woman has noted a nodule beneath the skin in her left groin since adolescence. On physical examination, the lesion has a 2-cm diameter and is nontender, soft, rubbery, and movable. Which of the following cell types is most likely to comprise this lesion? A Adipocyte B Endothelial cell C Fibroblast D Skeletal muscle E Smooth muscle

A The most common neoplasm is a lipoma, composed of mature adipocytes. They are slow growing and rarely produce problems. Endothelial cells are found in hemangiomas, which are common but firmer. In the groin region, they are likely to be on the skin surface. Fibroblasts may be found in reactive lesions such as fibromatoses, which are firm and immovable. Smooth muscle cells form firm leiomyomas. Neoplasms resembling skeletal muscle (rhabdomyomas) are quite rare.

5 A 23-year-old primigravida notes decreased fetal movement, and a screening ultrasound at 18 weeks' gestation shows decreased fetal size. A stillborn is delivered at 25 weeks' gestation. At autopsy, a radiograph shows marked osteopenia and multiple bone fractures. Mutational analysis of fetal cells is most likely to show an abnormality involving which of the following genes? A COL1A1 B EXT C FGFR3 D FBN1 E HGPRT F RB

A The stillborn has evidence for the type II lethal variant of osteogenesis imperfecta, with a defect in collagen 1 formation leading to multiple fractures in utero with long bone shortening. EXT mutations are seen in cases of hereditary and sporadic osteochondromas. The FGFR3 gene is implicated in cases of achondrogenesis with long bone shortening, but without osteopenia and fractures. Fibrillin-1 (FBN1) gene mutations are seen with Marfan syndrome, which severely affects the cardiovascular system, including aortic dissection. Hypoxanthine-guanine phosphoribosyltransferase (HGPRT) is an enzyme in the purine salvage pathway, and is associated with the X-linked Lesch-Nyhan syndrome with hyperuricemia. The tumor suppressor gene RB is associated with many cases of osteosarcoma.

23 A 38-year old healthy man notes occasional pain in his right index finger when using a hammer. On palpation of the right proximal phalanx there is point tenderness. A radiograph shows a 1-cm oval lucency in this phalanx, with a surrounding rim of bright, radiodense bone. The microscopic appearance of the excised lesion is shown in the figure. Which of the following cells is most likely to have given rise to this lesion? A Chondrocyte B Giant cell C Macrophage D Osteoblast E Plasma cell F Fibroblast image

A This is an enchondroma. Cartilaginous lesions may be benign when they occur peripherally (hands and feet) and are localized, but a low-grade chondrosarcoma is more likely in a central location. The risk for malignancy is higher with multiple enchondromas (Ollier disease or Maffucci syndrome with mutations in isocitrate dehydrogenase genes). Giant cells are seen in many mass lesions of bone, but particularly in giant cell tumors and aneurysmal bone cysts occurring in larger bones. Macrophages may increase in lysosomal storage diseases such as Gaucher disease that involve bone marrow. Osteoblasts may be seen in an osteoid osteoma, which is more likely to occur at a younger age and produce pain at night. Plasma cells in myeloma are unlikely to occur at this age or at this site. Fibroblasts may be seen in localized fibrous lesions such as fibrous cortical defects that involve long bones.

7) Which of the following statements correctly pair the skeletal muscle relaxant with its main mechanism of action? (Check all that apply) A) Succinylcholine - activates Nm receptors B) Mivacurium - inhibits acetylcholine synthesis in motor neurons C) Cisatracurium - blocks Nm receptors D) Dantrolene - blocks Ca++ channels in the skeletal muscle membrane E) Botulin toxin - inhibits acetylcholine release from cholinergic terminals

A, C, E

Point mutation in gene located on the short arm of chromosome 4

Achondroplasia

a fully humanized monoclonal IgG1 antibody directed against TNF-alpha. It has no foreign protein components. It is given by subcutaneous injection every 14 days using a pen delivery system. It binds to soluble and membrane -bound TNF- alpha.

Adalimumab

• Uses: Ankylosing spondylitis, Crohn's disease, Psoriasis with arthropathy, Rheumatoid arthritis • Mechanism of Action: • Blocks the interaction of TNF-alpha with TNF-alpha receptors on cell surfaces • Reduces ESR • Reduces serum IL-6 • Reduces metalloproteinases MMP-1 and MMP-3 • Patients can self-administer single doses subcutaneously every 14 days • Has a half-life of 2 weeks. This is increased by methotrexate ( 29-45%) • Black Box Warning: TB, invasive fungal infections, other opportunistic infections • Adverse effects: Exacerbates CHF, serious infections, SLE-like syndrome

Adalimumab

increases the half life of probenecid.

Allopurinol increases the half life of probenecid. Probenecid increases the clearance of alloxanthine thus a higher dose of allopurinol is required if the patient is on both drugs.

Pancuronium, Vecuronium, Rocuronium

Aminosteroids (non-polarizing blockers)

This is the ONLY biological which interferes with IL-1

Anakinra

• Recombinant form of Human IL-1 receptor antagonist. Naturally occurring anti-inflammatory cytokine but the drug is produced in E. Coli. • It has a short half-life and must be given daily • USES: Rheumatoid Arthritis • Mechanism of Action: Binds to and antagonizes IL-1 receptors and thus prevents IL-1 from acting on its receptors to produce inflammatory cytokines • Adverse Effects: Headache. GI Distress, injection site reaction is strong • Serious adverse effects: Infections- Decrease in White Cell Counts, antibody formation

Anakinra

15) A 67-year-old man required intubation and mechanical ventilation for management of respiratory failure resulting from severe emphysema. The patient was agitated, attempting to sit up in bed and reach for his endotracheal tube. Lorazepam was give IV for sedation and the patient got drowsy but motor restlessness was only marginally improved. A muscle relaxant was given IV. The patient relaxed as paralysis developed but 5 minutes later his heart rate was 160 bpm. Which of the following drugs most likely caused this adverse effect? A) Botulinum toxin B) Dantrolene C) Succinylcholine D) Tubocurarine E) Cisatracurium

Answer: D (Katzung, pp 438, Brunton, pp 226) Agitation is a frequent management problem in patients who are intubated and artificially ventilated. Benzodiazepines are useful agent for controlling anxiety and agitation. They also cause anterograde amnesia (a quite useful property in this setting) but have no analgesic properties. When sedating drugs are not fully effective, a muscle relaxant is added. Tubocurarine is a neuromuscular blocking drug whose action last more than 30 minutes. It is used because of effectiveness and low cost. The tachycardia that can appear after the administration of tubocurarine can be due either to histamine release or to blockade of nicotinic receptors of autonomic ganglia or, most likely, to both. A) Botulinum toxin is not used to limit movements in an intubated patient. It action would last for months. B) Dantrolene is a spasmolytic drug. It can reduce spasticity but is unable to cause neuromuscular paralysis when given at therapeutic doses. Therefore it is not used to limit movements in an intubated patient. C) Succinylcholine can cause stimulation of both nicotinic receptors of autonomic ganglia and cardiac muscarinic receptors. Therefore it would tend to cause bradycardia which is only partially counteracted by its slight tendency to cause histamine release. E) Cisatracurium has no effect on nicotinic receptors of autonomic ganglia and has negligible histamine releasing properties.

9) Which of the following is the earliest sign of the action of nondepolarizing neuromuscular blocking drugs? A) Difficulty in breathing B) Inability to rise the arm C) Inability to walk D) Fecal incontinence E) Diplopia

Answer: E (Katzung, pp 436, Brunton, pp 225) The skeletal muscle paralysis induced by nondepolarizing neuromuscular blocking drugs follows a sequence that is related to the innervation of the skeletal muscles. Muscles that are small, rapidly moving and richly innervated (extrinsic eye muscles, small muscles of the face and pharynx) are affected first. The paralysis of the extrinsic eye muscles causes a lack of parallelism of the visual axes of the eyes which leads to double vision. A) Respiratory muscles are the last to be paralyzed, so difficulty in breathing is not an early sign of the drug-induced paralysis. B, C) These sings indicate that muscles of the limbs are affected. As a rule, large muscles are paralyzed after small muscles. D) Fecal incontinence can occur because of the paralysis of the external anal sphincter (which is a striated muscle), but it occurs later, after the paralysis of rapidly moving muscles. 10) Answer: A

26 A 13-year-old, previously healthy boy has had pain in the right leg for the past month. There is no history of trauma or recent illness. On physical examination, there is warmth and tenderness to palpation of the right lower thigh anteriorly, and the circumference of the right thigh is slightly larger than that of the left. His temperature is 39° C. A radiograph of the right leg shows a 6-cm expansile mass in the diaphyseal region of the right lower femur that extends into the soft tissue and is covered by layers of reactive bone. A biopsy of the mass is done, and microscopic examination shows sheets of closely packed primitive cells with small, uniform nuclei and only scant cytoplasm. Karyotypic analysis of the tumor cells shows a t(11;22) translocation. What is the most likely diagnosis? A Chondrosarcoma B Ewing sarcoma C Giant cell tumor D Metastatic carcinoma E Osteosarcoma F Plasmacytoma 410

B

3. Decreasing B and T lymphocytes in treating rheumatoid arthritis has been shown to improve the condition. Which of the following drugs are able to do this by inhibiting pyrimidine synthesis? A) Colchicine B) Leflunomide C) 5-Fluoruracil D) Febuxostat E) Adalimumab

B

5. A 68-year-old woman had a stroke which resulted in combination of hypertonia and paralysis. Stoke is the most common cause of upper motor neuron syndrome that follows damage to the direct and indirect descending corticospinal fibers. Spasticity develops a few days or weeks after the stroke. What treatment could be used locally in flexor muscles to prevent acetylcholine from being released from its vesicles and reduce hypertonia because of post-stroke spasticity? A) An antispastic drug acting directly on muscle, dantrolene. B) A focal antispastic drug acting directly on the neuromuscular junction, botulinum toxin. C) A central acting muscle relaxant, diazepam. D) A centrally acting antispastic drug, baclofen. E) A centrally acting antispastic drug, tizanidine.

B

50 A 47-year-old man has had dull, constant pain in the midsection of the right thigh for the past 4 months. On physical examination, there is pain on palpation of the anterior right thigh, which worsens with movement. The right thigh appears to have a larger circumference than the left thigh. A radiograph of the right upper leg and pelvis shows no fracture, but there is an ill-defined soft-tissue mass anterior to the femur. MRI shows a 10 × 8 × 7 cm solid mass deep to the quadriceps, but it does not involve the femur. Karyotypic analysis of tumor cells reveals t(12;16)(q13;p11) with amplification of MDM2 gene. What is the most likely diagnosis? A Chondrosarcoma B Liposarcoma C Metastatic adenocarcinoma D Nodular fasciitis E Osteosarcoma F Rhabdomyosarcoma

B A large, deep soft-tissue mass suggests cancer, most likely a sarcoma. Liposarcomas are located in deep soft tissues, can be indolent, and can reach a large size. They are the most common sarcomas of adulthood. Chondrosarcomas can be seen over a wide age range, but they arise within bone. Carcinomas are far more common in adults than sarcomas, and can metastasize, but such a large lesion in soft tissue is unlikely to be a metastasis. Nodular fasciitis is a reactive fibroblastic lesion of young adults, usually on the upper extremities and trunk, and can develop several weeks after local trauma. Osteosarcomas generally occur in individuals younger than 20 years and typically arise in the metaphyseal region of long bones. Rhabdomyosarcoma occurs in children and is most often a tumor of the head and neck, genitourinary tract, or retroperitoneum.

28 A 75-year-old woman has experienced increasing dull but constant pain in the back, right chest, left shoulder, and left upper thigh for the past 6 months. She has now developed a sudden, severe, sharp pain in the left thigh. On physical examination, she has intense pain on palpation of the upper thigh, and the left leg is shorter than the right. A radiograph of the left leg shows a fracture through the upper diaphyseal region of the femur in a 5-cm lytic area that extends through the entire thickness of the bone. A bone scan shows multiple areas of increased uptake in the left femur, pelvis, vertebrae, right third and fourth ribs, upper left humerus, and left scapula. Laboratory studies show serum creatinine, 0.9 mg/dL; total protein, 6.7 g/dL; albumin, 4.5 g/dL; total bilirubin, 1 mg/dL; AST, 28 U/L; ALT, 22 U/L; and alkaline phosphatase, 202 U/L. What is the most likely diagnosis? A Hyperparathyroidism B Metastatic carcinoma C Multiple myeloma D Osteochondromatosis E Paget disease of bone F Polyostotic fibrous dysplasia

B An elevated alkaline phosphatase level in an older adult should raise the suspicion of bone metastases, particularly when there is a "pathologic" fracture resulting from a bone lesion, rather than a fracture from trauma. Likely primary sites include the breast (in women), prostate (in men), lung (in smokers), kidney, and thyroid. Hyperparathyroidism can lead to osteitis fibrosa cystica with lytic lesions that are usually small, involve just cortex, and appear first in phalanges. Multiple myeloma can produce lytic bone lesions, but the patient's serum gamma globulin level is not elevated. Osteochondromas are exostoses and do not produce lytic bone lesions. Paget disease of bone is characterized by osteolysis coupled with bone formation, but without lytic lesions. Fibrous dysplasia coupled with café-au-lait spots on skin and with endocrinopathies is known as McCune-Albright syndrome; this is a rare condition that occurs in young girls.

51 A 26-year-old man is struck in the left arm by a swinging steel beam at a construction site. On physical examination, a 4-cm area of the lateral upper left arm exhibits swelling and redness with pain on palpation. A radiograph of the left arm shows no fracture. Three weeks later, there now is a 2-cm painful, well-circumscribed, subcutaneous mass at the site of the original injury. A radiograph shows a solid soft-tissue mass. Which of the following lesions is most likely to be present in this man? A Lipoma B Nodular fasciitis C Organizing abscess D Pleomorphic fibroblastic sarcoma E Superficial fibromatosis

B Nodular fasciitis is a reactive fibroblastic proliferation that is seen in the upper extremities and trunk of young adults, sometimes occurring after trauma. A lipoma is a common benign soft-tissue tumor that is not painful and does not follow trauma. A contusion is unlikely to lead to abscess formation because there is no disruption of the skin to allow entry of infectious agents. Pleomorphic fibroblastic sarcoma 423is most likely to arise in the retroperitoneum and deep soft tissues of extremities in older adults. A superficial fibromatosis is a deforming lesion of fascial planes that develops over a long period, and the most common is a Dupuytren contracture involving the palm of the hand.

52 A 57-year-old woman has noticed increasing deformity and difficulty with movement involving her left hand over the past 6 months. On physical examination, there is a contracture involving the third digit of her left hand that prevents her from fully extending this finger. A firm, hard, cordlike 1 × 3 cm area is palpable beneath the skin of the left palm. Microscopically, which of the following is most likely to be seen in greatest abundance composing this lesion? A Atypical spindle cells B Collagen C Dystrophic calcification D Granulation tissue E Lipoblasts image

B The patient has superficial fibromatosis that has produced a lesion best known as a Dupuytren contracture. These lesions contain mature fibroblasts surrounded by dense collagen. A hard, firm lesion of this size is unlikely to be malignant. Though some fibrobastic cells are present with a spindle shape, they are not numerous and generally not significantly atypical. Dystrophic calcification occurs in necrotic tissues; it is not commonly a localized mass. Granulation tissue from an injury would give rise to a stable scar without such severe retraction. Lipoblasts are seen in a liposarcoma, which is more likely to arise in deep soft tissues, such as thigh or retroperitoneum.

17 A 7-year-old boy sustained an open compound fracture of the right tibia and fibula in a fall from a barn loft to the floor below. On physical examination, the lower tibia and fibula can be seen protruding from the lower leg. The fracture is set by external manipulation, and the skin wound is sutured, but nothing more is done. One year later, he continues to have pain in the right leg, and a draining sinus tract has developed in the lateral lower right leg. A radiograph of the lower right leg is now most likely to show which of the following? A Cortical nidus with surrounding sclerosis B Involucrum and sequestrum C Osteolysis with osteosclerosis D Soft-tissue hemorrhage and swelling E Tumor mass with bony destruction

B This patient has chronic osteomyelitis. The most likely sequence of events is the occurrence of a compound fracture that became infected by direct extension of bacteria into the bone. The subsequent care for this patient was inadequate, and he developed chronic osteomyelitis. Infection of the bone and the associated vascular compromise caused bone necrosis, giving rise to a dead portion of bone, called sequestrum. With chronicity, a shell of reactive new bone, called involucrum, is formed around the dead bone. A nidus with surrounding sclerosis suggests an osteoid osteoma. Osteolysis and osteosclerosis are features of bone remodeling with Paget disease of bone. Soft-tissue hemorrhage and swelling should be minimal and resolve soon after the fracture is stabilized. A mass suggests a malignancy, and the most common neoplasm to develop in a sinus tract draining from osteomyelitis is squamous cell carcinoma, but this is uncommon.

45 A 48-year-old man has had increasing pain in the left knee for the past 4 years, but the pain has become worse in the past week. On physical examination, the left knee is slightly swollen and warm to the touch. The cell count of a joint aspirate shows increased neutrophils. The figure shows a smear preparation of the fluid examined with polarized light microscopy (arrow in axis of red compensator). The patient experiences reduced knee joint mobility over the next 5 years. He also develops congestive heart failure, diabetes mellitus, and hepatic cirrhosis. Which of the following laboratory findings is most characteristic of this disease process? A ANA serologic titer of 1:1024 B Ferritin level in serum of 7245 ng/mL C Rheumatoid factor serologic titer of 1:512 D Serologic test for syphilis (STS) positive E Uric acid level in serum of 15.8 mg/dL

B This patient shows evidence of hemochromatosis-caused skin pigmentation, heart failure, diabetes, and cirrhosis. The figure shows negatively birefringent (blue) rhomboidal calcium pyrophosphate crystals that have been deposited in the articular matrix. In progression of the disease, the crystals can seed the joint space and give rise to pseudogout, or calcium pyrophosphate dihydrate deposition (CPDD) disease, also called chondrocalcinosis. CPDD can be primary (hereditary) or, more commonly, secondary to various systemic diseases, such as hemochromatosis or, in the elderly, secondary to preexisting joint damage from other conditions. In most autoimmune diseases with a positive ANA result, such as systemic lupus erythematosus, there are arthralgias, but no arthritis, and little or no joint swelling, destruction, or deformity occurs. Rheumatoid arthritis tends to be recurrent and causes progressive joint deformities, typically of the hands and feet. Congenital syphilis can produce periosteitis and osteochondritis with bone deformities; tertiary syphilis in adults can produce gummatous necrosis with joint destruction or loss of sensation, particularly in the lower extremities, leading to repeated trauma that deforms joints (Charcot joint). Some cases of gouty arthritis are accompanied by hyperuricemia; gouty arthritis tends to manifest as acute attacks in older individuals.

19) A 54-year-old male, who has been suffering amyotrophic lateral sclerosis for one year, complained of generalized muscle spasm. His physician prescribed diazepam to reduce spasticity. Which of the following statements best explain the mechanism of the spasmolytic effect of diazepam in this patient? A) Blockade of calcium release from the sarcoplasmic reticulum B) Blockade of acetylcholine release from motor nerve C) Depolarization blockade of Nm receptors D) Facilitation of GABA actions in the spinal cord E) Activation of GABAB receptors in the spinal cord

Benzodiazepines facilitate the action of GABA in the CNS by increasing the affinity of GABAA receptors for GABA. Their action in reducing spasticity however seems at least partly mediated in the spinal cord because they are effective also in patients with cord transection. A) This would be the mechanism of action of dantrolene. B) This would be the mechanism of action of botulinum toxin. C) This would be the mechanism of action of succinylcholine. E) This would be the mechanism of action of baclofen.

Tubocurarine, Mivacurium. Cisatracurium

Benzylioquinolines (non-polarizing blockers)

5) Which of the following drugs can antagonize the end plate depolarizing effect of succinylcholine? A) Baclofen B) Carbamazepine C) Tubocurarine D) Dantrolene E) Haloperidol F) Tizanidine

C

9. A 35-year-old woman presented with headache, palpitations, anxiety, flushing, hypertension, and postural hypotension. She was going to have major surgery and had been diagnosed with pheochromocytoma. It is important that the drugs used to support surgery do not aggravate the cardiovascular system. Which non-depolarizing drug could be safely used in this patient during surgery to relax muscles? A) Succinylcholine B) Pancuronium C) Cisatracurium D) Baclofen E) Tubocurarine

C

A 34-year-old man had internal surgery and a depolarizing drug was administered to support tracheal intubation prior to anesthesia. This drug worked to contract and then relax the muscle and caused what to happen at the neuromuscular junction? 2. This was the first time the patient (see above) ever had any surgery and he presented with acidosis, hypoxia, hyperkalemia, increased temperature, caused tachycardia and muscle rigidity. Isoflurane was the volatile anesthetic used. Succinylcholine was used for tracheal intubation. IV dantrolene was quickly administered. How did this drug work to reverse the symptoms presented by this patient? A) Increase release of Ca 2+ B) Block Ca receptors C) Prevent Ca 2+ release D) Inhibit ACh release E) Block ACh receptors F) Increase Ach levels

C

24 A 30-year-old woman has experienced pain in the area of the left knee for 1 month. On physical examination, there is tenderness to palpation of the distal left thigh and knee. The area is firm, but there is no erythema or warmth. A radiograph of the left leg shows a 7-cm mass in the distal femoral epiphyseal area, with a "soap bubble" appearance. Microscopic examination of a curettage specimen of the lesion shows the findings in the figure. The lesion recurs in the next year; it is excised and does not recur again. What is the most likely diagnosis? A Chondrosarcoma B Enchondroma C Giant cell tumor D Osteitis fibrosa cystica E Osteoblastoma F Plasmacytoma

C Giant cell tumors typically arise in the epiphyses of long bones of individuals 20 to 40 years old; there is a slight female predominance. The tumors may recur after curettage. Although most are histologically and biologically benign, with multinucleated cells in a stroma predominantly composed of spindle-shaped mononuclear cells as shown in the figure, in rare cases, a sarcoma can arise in a giant cell tumor of bone. Chondrosarcomas are typically larger destructive lesions. Enchondromas are most often peripheral skeletal lesions involving the metaphyseal region of small tubular bones of the hands and feet. An osteoblastoma usually involves the spine. Osteitis fibrosa cystica is a complication of hyperparathyroidism. A plasmacytoma composed of neoplastic plasma cells is most often one lesion of multiple myeloma, more likely to occur in older adults.

41 A 27-year-old man develops acute pain and swelling of the left knee 5 days after an episode of urethritis. On physical examination, the left knee is swollen, warm, and tender to the touch. No other joints are affected. Laboratory examination of fluid aspirated from the left knee joint shows numerous neutrophils. A Gram stain of the fluid shows with gram-negative intracellular diplococci. No crystals are seen. Which of the following infectious agents is most likely responsible for his condition? A Borrelia burgdorferi B Haemophilus influenzae C Neisseria gonorrhoeae D Staphylococcus aureus E Treponema pallidum image

C Gonorrhea should be considered the most likely cause of an acute suppurative arthritis in sexually active individuals; in some cases multiple joints can be involved. In men, a urethritis may occur with gonorrheal infection. Borrelia burgdorferi causes Lyme disease, characterized by chronic arthritis that may mimic rheumatoid arthritis. Haemophilus influenzae is a short, gram-negative rod that can cause osteomyelitis in children. Staphylococcus aureus is the most common cause of osteomyelitis, but the Gram stain would show gram-positive cocci. Treponema pallidum infection, also a sexually transmitted disease, can lead to syphilitic gummas in the tertiary phase of syphilis that may produce joint deformity. There is no preceding urethritis, however. Tertiary syphilis may be preceded years earlier by a primary syphilitic chancre.

4 A 14-year-old girl who was normal at birth now has bilateral hearing loss. Audiometry indicates bilateral mixed conductive and sensorineural hearing loss. CT scan of the head shows maldevelopment of both middle ears with deficient ossification. Further history indicates that her dentist has tried various whiteners to diminish the yellow-brown color of her teeth, which have a slight bell-shaped appearance. The optometrist noted that her sclerae have a peculiar steel-gray color, and her vision is 20/40. At age 30 years, she falls and fractures the left femur. A radiograph shows that the femur is osteopenic. Bone densitometry reveals osteopenia of all measured sites. Which of the following molecular mechanisms is most likely to produce these findings? A Deficient hypoxanthine-guanine phosphoribosyltransferase (HGPRT) activity B Diminished osteoprotegerin binding to macrophage RANK receptor C Failure of type I collagen formation by osteoblasts D Fibroblast growth factor receptor 3 inhibition of cartilage proliferation E Increased interleukin-6 production by osteoblasts F Reduced number of vitamin D receptors 406

C The patient has osteogenesis imperfecta, most likely type I. Type II is the perinatal lethal form, which causes death in utero, at birth, or shortly thereafter. Type III is seen in children and adults and is more severe than type I. Type IV is difficult to distinguish from type III. Osteogenesis imperfecta causes osteopenia ("brittle bones") and predisposes to fractures. Patients often have "blue" sclerae, dental abnormalities, and progressive hearing loss. Absence of HGPRT is an X-linked disorder known as Lesch-Nyhan syndrome and characterized by hyperuricemia. Decreased binding of osteoprotegerin to the macrophage RANK receptor is part of the mechanism of osteoporosis. Mutations in the fibroblast growth factor receptor 3 (FGFR3) gene can be seen in achondroplasia. Increased interleukin-6 production by osteoblasts occurs in Paget disease of bone and with postmenopausal decrease in estrogen, causing increased bone loss. Decreased absorption of vitamin D in the small intestine leads to rickets in children; glucocorticoids can decrease receptor expression in many tissues, including intestine.

29 A 78-year-old woman has had a constant, dull pain in her back that has persisted for more than a month. She is in no acute distress. On physical examination, there are no abnormal findings. Laboratory findings include creatinine, 0.9 mg/dL; urea nitrogen, 17 mg/dL; total protein, 6.8 g/dL; albumin, 4.2 g/dL; total bilirubin, 0.8 mg/dL; AST, 25 U/L; ALT, 29 U/L; calcium, 10.8 mg/dL; phosphorus, 2.3 mg/dL; and alkaline phosphatase, 228 U/L. What is the most likely cause of hypercalcemia? A Chondrosarcoma B Gouty arthritis C Metastatic carcinoma D Osteoarthritis E Osteoporosis F Paget disease of bone

C The prevalence of cancer increases with age. In the absence of other findings, a metastatic carcinoma involving bone should be suspected. In addition, parathyroid hormone-related peptide elaborated by neoplasms can be a cause for hypercalcemia of malignancy. Chondrosarcoma can occur over a wide age range, but these focal lesions involving bone are not likely to elevate serum calcium. Metastases to bone are far more common than primary bone malignancies. Biliary tract obstruction or infiltrative diseases of the liver are usually the cause for an elevated alkaline phosphatase of liver origin. Gouty arthritis may be accompanied by some local bone destruction, but without a marked increase in serum calcium and the lesions are typically in peripheral joints. Osteoarthritis produces pain, usually involving weight-bearing joints or hands, but has no abnormal laboratory markers. Although there is accelerated bone loss with osteoporosis, alkaline phosphatase and serum calcium are normal. Paget disease of bone is associated with an increased alkaline phosphatase, but not hypercalcemia.

primary (75%), or secondary (to exostosis, chondroma, chondroblastoma) middle to old age, in the central skeleton. lobulated, translucent mass, composed of malignant hyaline and myxoid cartilage with areas of necrosis & calcification. x-ray: radiolucent mass; bone destruction, and mottled calcification Well differentiated (low-grade, small) tumors show late distant spread & good prognosis --up to 90%-5 yr. surv.

CHONDROSARCOMA

malalignment, comminution, inadequate immobilization, infection, osteoporosis, systemic diseases, ...all may lead to delayed union, non-union or pseudoarthrosis

Causes of poor healing: malalignment, comminution, inadequate immobilization,

site: epiphysis; age: children; hyaline cartilage, with embryonic chondroblasts & osteoclast-like giant cells. malignant potential

Chondroblastoma an osteochondroma would be found at the metaphysis of long bones

composed of hyaline cartilage, sporadic or multiple. Ollier's disease = nonfamilial & Maffucci's synd. = familial enchondromatosis +hemangiomas.

Chondroma (enchondroma)

cartilage, fibrous & myxomatous tissues; adolescents; metaphyseal.

Chondromyxoid Fibroma:

Colchicine (an antimitotic agent) • Mechanism of action • It inhibits cell division (anti-mitotic) • It acts by binding to tubulin preventing the formation of microtubules which leads to inhibition of leucocyte migration into joints as their mobility is compromised. Besides decreasing the movements of neutrophils, it blocks lipoxygenase enzymes thus inhibiting the synthesis of the inflammatory leukotrienes(especially leukotrieneB4.). Intracellular trafficking is compromised too as it also depends on intact microtubular protein Dramatic relief from pain is achieved.

Colchicine

It is rapidly absorbed after oral administration and plasma levels peak 0.5- 2.0 hr. • It is sometimes given IV. • It undergoes extensive enterohepatic recirculation. Drug secretion into the bile is mediated by the MDR protein pump. Drugs which inhibit liver MDR protein such as cyclosporine and verapamil significantly increase the fraction delivered to and remaining in the systemic circulation as it will not be delivered into the gut for excretion . The dose must be lowered if it given concurrently with any drug known to inhibit MDR activity. • causes serious GIT toxicity, N, V, D, and pain. reduces ADH effects as the water channels cannot move into place due to compromised tubulin.

Colchicine

Lipomas & Liposarcomas Fibroma, Fibrosarcoma, Benign & Malignant Fibrous Histiocytomas. Nodular Fasciitis & Fibromatosis. Leiomyomas & Leiomyosarcomas. Rhabdomyoma & Rhabdomyosarcoma. Synovial Sarcoma..

Common Soft Tissue Tumors

3) What will be the likely EMG findings in this patient? A) Short-duration low amplitude action potentials B) Silence C) Dive-bomber sounds D) Decreased response to repetitive stimulation

Correct answer is A. It takes many muscle units to produce normal action potentials on EMG. As muscles get destroyed in MD, the diminished number of fibers and units do not generate as large or as long an action potential; so the action potentials on this EMG will be of lower height (amplitude) and shorter width (duration).

5) A 48-year-old man sees his physician because of joint pains and rash that have been present for a few days. He has noted some burning with urination as well. About a month ago he experienced a diarrhea illness that lasted several days and gradually resolved completely. Upon examination he has a slightly swollen, warm and tender right wrist. His left elbow is tender and warm and he cannot fully extend it. He has raised erythematous macules on the soles of his feet. He has peri-urethral inflammation extending outward on the glans. Which of the following is the most likely diagnosis? A) Reactive arthritis B) Gonococcal arthritis C) Rheumatoid arthritis D) Gout E) Pseudogout

Correct answer is A. The clues here are oligoarthritis appearing after a diarrheal illness (Salmonella, Shigella, Campylobacter, Yersinia) associated with urethritis, uveitis, rash, balanitis.

7) A 26-year-old woman presents to the physician with pain and swelling in both wrists for the last 2-3 weeks. Upon examination she has redness, warmth and swelling and tenderness over both wrists and tenderness of the MP joints of both hands. ESR is 45 (normal up to 20). Which of the following is the most likely diagnosis? A) Rheumatoid arthritis B) SLE C) Fibromyalgia D) Osteoarthritis E) Pseudogout

Correct answer is A. This is a distal symmetrical polyarthritis and she is a young female, making the most likely diagnosis RA.

4) A 38-year-old man suffers from chronic back pain for the last 5 years and has been prescribed hydrocodone, an oral opioid, to manage his pain. His medical evaluation has revealed no anatomic or physiologic cause for his back pain. He frequently requests early refills of his medication because of breakthrough pain, but has not been adherent to other recommendations for pain management including exercise and mindfulness meditation training. Which of the following is most important in deciding to discontinue prescribing this patient's narcotic medication? A) Pain and function are not improved B) The medication keeps getting stolen C) The patient is developing constipation D) Tolerance is developing E) The medication makes it easier to deal with people

Correct answer is A: In prescribing narcotics for chronic pain, the most important indication for continuing the narcotic is that the patient is getting pain relief and is functioning better with the medication. If neither of these two objectives is being met, the medication is not working and should be withdrawn (slowly) and other pain management options considered. Request for early refill is an indication that the medication may not be working. Patient's often hide their early refill requests in "stories" about the medication getting stolen or lost.

1) A 36-year-old woman reports to her family physician with acute low back pain that started two days ago at work. She is a secretary for a local businessman and she hates her job. She has tenderness to palpation over her right lower paraspinous muscles, but no other findings on physical examination. Which of the following is most important to obtain in her first visit? A) A history of stress and anger B) A lumbar spine x-ray C) A sedimentation rate D) An MRI of her lumbar spine E) A referral to a counselor

Correct answer is A: Remember to get a stress and anger history at the first visit so that you can determine if this may be on your differential diagnosis for non-specific back pain. X-rays and lab work will only be done if the patient falls outside that 85% category of non-specific back pain and you will get those tests only if you suspect injury, inflammatory disease, cancer, etc.

5) A 15-year-old boy is brought to the pediatrician because he is failing gym class. The teacher feels he is unmotivated; his father is concerned because he no longer wants to ride his bike as he's fallen a few times. The physician notes that the boy has an exaggerated lumbar lordosis and trouble climbing up to sit on the exam table. Which of the following is the likely diagnosis? A) Duchenne muscular dystrophy B) Becker muscular dystrophy C) Limb-girdle muscular dystrophy D) Myotonic muscular dystrophy

Correct answer is B. Duchenne manifests much earlier in life. This actually might be limb girdle MD, which would be included in the differential diagnosis. Becker and limb girdle MD occur with close to the same frequency and because this patient is a boy, you cannot so easier decide which MD this is. So, if you narrowed the answer down to these two and your supporting and refuting arguments sound like this explanation, then you know your MDs. Test questions will not be ambiguous like this.

1) A 3-year-old boy is brought to his pediatrician because he has trouble climbing stairs and just does not seem to be able to keep up with other toddlers physically at pre- school. Developmental milestones are otherwise normal. Upon physical examination, his calves appear extremely well developed. The physician places the child on the floor and coaxes him to get up by offering a toy. The child pushes himself to onto his hands and feet and then uses his hands to climb up his thighs to get into a standing position. What is the likely diagnosis? A) Limb-girdle muscular dystrophy B) Duchenne muscular dystrophy C) Becker muscular dystrophy D) Myotonic dystrophy E) Metabolic myopathy

Correct answer is B. The early age of onset of symptoms, calf pseudohypertrophy and Gower maneuver are clues in this vignette. Becker MD tends to occur in later childhood as does limb-girdle MD. This patient does not demonstrate muscle stiffness or myotonic muscle contractions of myotonic MD, and he is too young for the onset of that disorder. Anyone can get a metabolic myopathy, but there is nothing in this vignette to suggest one.

6) Dr. Selfridge relates a story of meeting a man in her preceptor's office during the first year of medical school about 38 years ago. She noticed that the man appeared in his 30s and had significant frontal balding. Her preceptor introduced her to this patient and told her to shake his hand, which she did. However, over the next 45 seconds, she was unable to retrieve her hand from the man's grip. What was this gentleman's likely diagnosis? A) Muscular dystrophy B) Myotonic dystrophy C) Metabolic myopathy D) Myotonia congenita

Correct answer is B. This man demonstrates myotonia during the handshake and his early frontal baldness is a clue in the vignette.

4) A 76-year-old woman reports that she had to stop deer hunting with her husband last fall because she could no longer climb the tree into their deer blind. She denies any pain. Upon physical examination she is noted to have significantly limited range of motion (ROM) in both hips to flexion and external rotation. Bilateral hip x-rays are ordered and they are abnormal. Which of the following findings are most likely seen on the patient's hip x-rays? A) Bony erosions B) Subchondral sclerosis C) Tophi D) Heberden's nodes E) Slipped capital epiphysis

Correct answer is B. This woman describes an arthritis that is associated with little or no pain and reduced ROM. Osteoarthritis pain does not correlate with severity of x-ray findings. X-ray findings of OA also include osteophytes, joint space narrowing, and sub - chondral cysts.

4) A 14-year-old girl is brought to the pediatrician for a gym excuse. She does not like gym class because it is hard for her. She has trouble playing dodge ball because she cannot move fast enough. She cannot raise the ball over her head to throw it. The physician notes that she has a great deal of difficulty getting onto the step up to the exam table. Examination confirms symmetrical proximal weakness. Which of the following is the most likely diagnosis? A) Duchenne muscular dystrophy B) Becker muscular dystrophy C) Limb-girdle muscular dystrophy D) Myotonic muscular dystrophy

Correct answer is C. Duchenne and Becker are X-linked and will only affect boys. In addition, this patient is demonstrating proximal and limb girdle weakness on history and physical examination. This vignette does not describe any characteristics of myotonia in this patient.

A 3-year-old boy is brought to his pediatrician because he has trouble climbing stairs and just does not seem to be able to keep up with other toddlers physically at pre- school. Developmental milestones are otherwise normal. Upon physical examination, his calves appear extremely well developed. The physician places the child on the floor and coaxes him to get up by offering a toy. The child pushes himself to onto his hands and feet and then uses his hands to climb up his thighs to get into a standing position. 2) For the previous patient, which of the following will be elevated? A) Dystrophin B) WBC C) Creatine kinase D) Creatinine

Correct answer is C. Dystrophin is absent in Duchenne MD patients. The WBC will be normal. Creatinine is a measure of renal function. Creatinine kinase is released into the blood any time there is breakdown of a muscle cell, which is part of the pathology of muscular dystrophies.

1) A 12-year-old boy develops acute swelling, pain and redness of his right knee. Upon examination, the right knee is painful even with the slightest movement. A CBC is performed and reveals a WBC of 18,000 with 80% neutrophils. Arthrocentesis results: Appearance: pale yellow, cloudy Gram stain: Gram positive cocci in clusters WBCs: 75,000/mm3, 85% neutrophils No crystals present String test: no string formed Culture: pending Which of the following is the next best step for managing this problem? A) Postpone antibiotic therapy until culture results return B) Prescribe oral antibiotic therapy C) Prescribe intravenous antibiotic therapy D) Prescribe etanercept for inflammation E) Refer to a rheumatologist for further work-up

Correct answer is C. This patient has septic arthritis. This requires immediate intervention, starting with IV antibiotics, in order to prevent joint destruction.

3) A 65-year-old woman is experiencing diffuse pain in her neck, shoulders and low back, which has come on gradually over the past 9 months. She has morning stiffness lasting for at least 2 hours and fatigue, malaise and loss of appetite. Upon physical examination she has widespread tenderness to palpation over her back and arm muscles. Her erythrocyte sedimentation rate is 110 (normal: <20). Which of the following is the most likely explanation for her pain? A) Rheumatoid arthritis B) Osteoarthritis C) Polymyalgia rheumatica D) Psoriatic arthritis E) Systemic lupus erythematosus

Correct answer is C. This woman has generalized pain, tenderness and "constitutional" symptoms (fatigue, malaise). Her ESR is VERY high. RA gives a symmetrical distal polyarthritis. Osteoarthritis is not a systemic disorder and affects hand joints, knees and hips. Psoriatic arthritis is associated with skin rash, nail signs and DIP arthritis. SLE is possible but this vignette does not provide enough criteria.

2) A 72-year-old man reports with constant low back pain of 3 months duration, present at night and not relieved by change in position. He notes that his appetite is diminished and his clothing has become loose. Past medical history is positive for occasional brief episodes of low back pain when he was working as a car mechanic between ages 20 and 65. Physical examination reveals a thin, tired appearing man. His spine is straight. He has midline tenderness to percussion over 2-3 vertebrae in the mid-thoracic region. Digital rectal examination reveals a stony hard, irregular and enlarged prostate. Which of the following is a high priority in this patient for this visit? A) Referral to a pain management center B) Referral to a pain counselor C) An imaging study of the spine D) An exercise regimen E) Stress management advice

Correct answer is C: With loss appetite and weight loss and unrelenting pain, as well as a rectal examination that suggests prostate cancer, this patient has metastatic cancer of his spine until proven otherwise and is a high priority patient for imaging of his spine.

3) A 45-year-old woman with a diagnosis of fibromyalgia completes a McGill Pain Questionnaire marks many of the terms as describing her pain. Which of the following terms from this questionnaire is considered an affective descriptor? A) Throbbing B) Lancinating C) Vicious D) Burning E) Continuous

Correct answer is C: the other terms are sensory or evaluative descriptors. In general, affective descriptors tend to "personify" the pain. For most of these descriptors, you could apply the adjectives to a person. However you would not use an evaluative descriptor for a human being (eg, a lancinating person, or a burning person).

2) A 45-year-old man "strains" his right knee during a recreational game of football. He applies ice and over 3 days his knee pain improves. Two weeks later he awakes in the middle of the night with excruciating right knee pain. He reports to the emergency department, where he is noted to have a red, hot and swollen right knee. Arthrocentesis is performed and the synovial fluid demonstrates white cells but there are no bacteria on gram stain. The synovial fluid is examined using polarized light microscopy. There are negatively birefringent crystals present. Which of the following best describes appropriate acute treatment for this gentleman? A) Intravenous antibiotic therapy B) Arthroscopy for anterior cruciate ligament repair C) Oral febuxostat D) A non-steroidal anti-inflammatory drug E) An intramuscular injection of gold salts

Correct answer is D. This patient has gout as a cause of his arthritis. The initial injury stated here is a typical distractor in this vignette. Only two drugs in the answer choices treat gout, febuxostat and NSAIDs. Febuxostat is a xanthine oxidase inhibitor and reduces serum uric acid over time. It does not work for an acute episode of gout.

7) An 8-year-old child is brought to the pediatrician because of complaints of pain that her parents believe are growing pains. The physician notes that both eyelids are pinkish purple and she has erythematous to purplish papules on the dorsum of her hands over the MP, PIP and DIP joints. Which of the following is the likely diagnosis? A) Polymyositis B) Polymyalgia rheumatica C) Systemic lupus erythematosus D) Dermatomyositis E) Scleroderma

Correct answer is D. This vignette describes a patient with musculoskeletal pain, and she has characteristic skin signs (heliotrope and Gottron's papules). If other physical examination findings were included in this vignette, they would include some muscular weakness. Remember that this condition is chronic and is associated with increased risk of visceral cancer.

6) A 19-year-old woman complains of back pain for several years, which her pediatrician has called "growing pains". Her buttocks are painful during flares. She has noted that her left hip has begun to hurt. All of her symptoms wax and wane and tend to be much better when she is active and moving, which has made it hard for her to sit through long days of college classwork. Upon examination, she has tenderness over both sacroiliac joints. She is unable to fully extend the left hip. Which of the following is the most likely diagnosis? A) Rheumatoid arthritis B) SLE C) Polymyalgia rheumatic D) Osteoarthritis E) Ankylosing spondylitis

Correct answer is E. The clues here are young age of onset pain in the back and buttocks that improves with movement, SI joint tenderness.

competitive non-polarizing agent, blocks nicotinic ACh

Curare

1. A 31-year-old woman consults a physician complaining that since she started therapy for her rheumatoid arthritis, she has developed night-blindness and notices light flashes occurring at random. Which anti-arthritic drug is she most likely to be taking? A) Colchicine B) Infliximab C) Ibuprofen D) Chloroquine E) Penicillamine

D

30 A 71-year-old man has experienced aching pain in the right knee, lower back, right distal fifth finger, and neck over the past 10 years. He notices that the joints feel slightly stiff in the morning, but this passes quickly. The pain is worse toward the end of the day. On physical examination, there is no joint swelling, warmth, or deformity. Some joint crepitus is audible on moving the knee. Laboratory studies show normal levels of serum calcium, phosphorus, alkaline phosphatase, and uric acid. What is the most likely cause of hypercalcemia? A Ankylosing spondylitis B Gouty arthritis C Multiple myeloma D Osteoarthritis E Pseudogout F Rheumatoid arthritis

D

A 34-year-old man had internal surgery and a depolarizing drug was administered to support tracheal intubation prior to anesthesia. This drug worked to contract and then relax the muscle and caused what to happen at the neuromuscular junction? 4. Hypothetically, we could of used a non-depolarizing drug with the right pharmacokinetics, and we might of avoided malignant hyperthermia. Why don't we use most current non-depolarizing drugs for intubation? A) Acetylcholine is increased in the neuromuscular junction. B) Acetylcholine is increased in the heart. C) Irritates the airways. D) The onset of action is not short enough and the duration of action is often too long. E) Increase acetylcholine in airway smooth muscle. F) Block histamine in airway smooth muscle.

D

The following 5 questions begin with this case study. 1. A 34-year-old man had internal surgery and a depolarizing drug was administered to support tracheal intubation prior to anesthesia. This drug worked to contract and then relax the muscle and caused what to happen at the neuromuscular junction? A) Blocked the nicotinic ion channel and kept it in a closed position. B) Bound to the nicotinic ion channel and then quickly came off the receptor so that it could be activated again. C) Kept the channel closed and prevented propagation of an action potential. D) Opened the nicotinic channel, kept it open, desensitized the channel and then blocked the ability to initiate an action potential. E) Kept the nicotinic channel open and allowed for continual contraction. F) Prevented the release of acetylcholine from vesicles in the presynaptic end of the neuron at the neuromuscular junction.

D

2 A 39-year-old man on vacation is involved in a skiing accident in which he sustains a right tibial diaphyseal fracture. The fracture is set with open reduction and internal fixation for proper alignment. What is the most likely function of osteoclasts present at his fracture site 1 week later? A Dividing mitotically B Elaborating cytokines C Forming collagen D Resorbing bone E Synthesizing osteoid

D Bone remodeling is a balance between osteoblastic and osteoclastic activity. The decoy protein osteoprotegerin (OPG) can bind RANK ligand to reduce its binding to RANK and reduce the nuclear factor kappa B transcription factor and diminish osteoclast activity. The WNT/β-catenin pathway can increase OPG production to tip the balance in favor of bone formation. Bone morphogenetic proteins oppose fibroblast growth factors to decrease chondrocyte proliferation. Matrix metalloproteinases generated from osteoclasts promote bone resorption.

7 A 2-year-old child has a history of multiple bone fractures with minor trauma. On examination he has hepatosplenomegaly and palsies involving cranial nerves II, VII, and VIII. Laboratory studies show pancytopenia. Radiographs reveal diffusely and symmetrically sclerotic bones with poorly formed metaphyses. Molecular analysis of his bone reveals a defect in production of carbonic anhydrase to solubilize hydroxyapatite crystal. He is treated with hematopoietic stem cell transplantation. Which of the following cells in his bones was most likely functionally deficient and replaced following transplantation? A Chondroblast B Chondrocyte C Osteoblast D Osteoclast E Osteocyte

D Osteopetrosis (Albers-Schönberg disease) is a rare bone disease resulting from mutations in genes that regulate osteoclast activity and bone resorption. Carbonic anhydrase generates the protons used by the H+-ATPase proton pump located on the osteoclast ruffled border. Marrow is reduced within the sclerotic bone, with subsequent extramedullary hematopoiesis in spleen and liver. Nerve compression in narrowed bony foramina leads to the palsies. The other cells listed are derived from mesenchymal stem cells, not hematopoietic stem cells. Osteopetrosis is the opposite of osteoporosis. Remember that in osteoporosis the osteoclast formation and action are enhanced when osteoblasts produce less osteoprotegerin and M-CSF, and increased RANK and RANKL interaction promote osteoclast differentiation and survival.

16 A 9-year-old boy has had pain in the area of the right hip for the past 3 weeks. On physical examination, his temperature is 38.2° C. There is swelling with marked tenderness to palpation in the area of the right hip, pain, and reduced range of motion. Radiographs of the pelvis and legs show areas of osteolysis and cortical erosion involving the femoral metaphysis, with adjacent soft-tissue swelling extending from the subperiosteal region, and apparent abscess formation. Which of the following organisms is most likely to produce these findings? A Haemophilus influenzae B Neisseria gonorrhoeae C Salmonella enterica D Staphylococcus aureus E Group B streptococcus

D Pyogenic osteomyelitis may arise from hematogenous dissemination of an infection. In children with no history of previous illnesses, Staphylococcus aureus is the most common causative organism. Haemophilus influenzae and group B streptococcal infections are most common in the neonatal period. Gonorrhea occasionally may disseminate and involve the bones (osteomyelitis) or joints (septic arthritis) of sexually active individuals. Salmonella infection involving bone is infrequent, except in patients with sickle cell anemia.

40 An 8-year-old boy complains of left leg pain for 3 days. On physical examination, his temperature is 38.9° C, and he exhibits irritability when his left leg is moved. A radiograph of the left leg shows changes suggesting acute osteomyelitis in the proximal portion of the left femur. Culture of the infected bone is most likely to grow which of the following organisms? A Hemophilus influenzae B Neisseria gonorrhoeae C Salmonella enterica D Staphylococcus aureus E Streptococcus pneumoniae

D Staphylococcus aureus is the most common cause of osteomyelitis in children and adults. Infections with H. influenzae may occur in children less than 2 years of age, but vaccination has reduced the incidence of such cases. Both organisms can cause congenital infections. Gonorrhea as a cause of acute osteomyelitis should be considered in sexually active adults, and may be passed to the fetus in the birth canal, but typically involve the eyes. Salmonella osteomyelitis is most characteristic of individuals with sickle cell anemia. Pneumococcal osteomyelitis is uncommon.

11 A 70-year-old man complains of right hip and thigh pain of 8 months' duration. On physical examination, he has reduced range of motion in both hips, but there is no tenderness or swelling on palpation. Radiographs of the pelvis and right leg show sclerotic, thickened cortical bone with a narrowed joint space near the acetabulum. Laboratory studies show a serum alkaline phosphatase level of 173 U/L, calcium of 9.5 mg/dL, and phosphorus of 3.4 mg/dL. The figure shows the representative microscopic appearance of his pelvic bone. Which of the following conditions is most likely to produce these findings? A Degenerative osteoarthritis B Hyperparathyroidism C Osteochondroma D Paget disease of bone E Vitamin D deficiency

D The mosaic pattern of lamellar bone in the figure is characteristic of osteitis deformans (Paget disease of bone). This disease has three phases. Early in the course, there is a lytic phase, followed by the more classic mixed phase of osteosclerosis and osteolysis, leading to the appearance of a "mosaic" of irregular bone. A sclerotic "burnt-out" phase then ensues. Elderly white people are most often affected, and the disease progresses over many years. Joints adjacent to affected bone manifest osteoarthritis with chronic pain from joint erosion. Osteitis fibrosa cystica is seen in hyperparathyroidism. An osteochondroma is a tumorlike projection of bone capped by cartilage that protrudes from the metaphyseal region of a long bone. Osteomalacia results in osteopenia from vitamin D deficiency in an adult.

36 A 30-year-old man has had cramping abdominal pain and bloody diarrhea for the past 4 days. On physical examination, there is diffuse tenderness on palpation of the abdomen. Bowel sounds are present. There are no masses and no organomegaly. A stool culture is positive for Shigella flexneri. The episode resolves spontaneously within 1 week after onset. Six weeks later, he has increasingly severe lower back pain. Physical examination now shows stiffness of the lumbar spine and tenderness of the sacroiliac joints. He is treated with nonsteroidal anti-inflammatory agents. Two months later, the back pain recurs, and he complains of redness of the right eye and blurred vision. Serologic testing for which of the following is most likely to be positive in this patient? A Borrelia burgdorferi B Chlamydia trachomatis C Epstein-Barr virus D HLA-B27 E Rheumatoid factor 412

D This patient developed enteritis-associated arthritis affecting the lumbar and sacroiliac joints several weeks after Shigella dysentery. He subsequently developed conjunctivitis and, most likely, uveitis. This symptom complex is a classic representation of a cluster of related disorders called seronegative spondyloarthropathies. This cluster includes ankylosing spondylitis, reactive arthritis, psoriatic arthritis, and enteropathic arthritis (as in this case). A common feature is a very strong association with the HLA-B27 genotype. Despite some similarities with rheumatoid arthritis, these patients invariably have a negative test result for rheumatoid factor. Urethritis caused by Chlamydia trachomatis can trigger reactive arthritis, another form of seronegative spondyloarthropathy. Such infection precedes the onset of arthritis, however. There is no relationship between infection with Borrelia burgdorferi, the causative agent of Lyme disease, and reactive arthritis in individuals testing positive for HLA-B27. Similarly, Epstein-Barr virus infection is not a trigger for these disorders.

38 A 15-year-old boy has been hospitalized multiple times since childhood as a result of painful abdominal crises. He has had pain in his right hip region for the past week. On physical examination, there is marked tenderness and swelling to palpation over the right hip. Laboratory studies show hemoglobin of 8.5 g/dL, hematocrit of 25.7%, platelet count of 199,900/mm3, and WBC count of 12,190/mm3. Examination of the peripheral blood smear shows sickled erythrocytes, and nucleated RBCs. A radiograph of the pelvis and right upper leg shows acute inflammatory changes in the femoral head and metaphysis of the right proximal femur. Which infectious agent is most likely responsible for these findings? A Group B streptococcus B Klebsiella pneumoniae C Mycobacterium tuberculosis D Salmonella enterica E Staphylococcus aureus

D Though Staphylococcus aureus infection is responsible for 80% to 90% of all cases of osteomyelitis in which an organism can be cultured. Salmonella osteomyelitis is especially common, however, in patients with sickle cell anemia. Group B streptococcal infections causing osteomyelitis are most common in neonates. Klebsiella pneumoniae osteomyelitis may rarely be seen in adults with urinary tract infections caused by this organism. Tuberculosis is a rare cause of osteomyelitis 421in adults who have had active pulmonary disease with dissemination, most likely because of a poor immune response.

It is the primary drug used for the treatment and prevention of malignant hyperthermia, a rare, life-threatening disorder triggered by general anesthesia

Dantrolene

a postsynaptic muscle relaxant that lessens excitation-contraction coupling in muscle cells. It achieves this by inhibiting Ca2+ ions release from sarcoplasmic reticulum stores by antagonizing ryanodine receptors

Dantrolene

RANKL antibody

Denosumab

approved for patients who have renal problems or cannot tolerate bisphosphonates

Denosumab

8. A 65-year-old man presented with severe spasticity due to traumatic brain and spinal cord injury. A drug was recommended for his spasticity that does not have CNS side effects. It reduces spasticity at the muscle level, but has to be monitored for liver damage regularly. What drug was most likely prescribed? A) Succinylcholine B) Pancuronium C) Baclofen D) Diazepam E) Dantrolene

E

A 68-year-old woman had a stroke which resulted in combination of hypertonia and paralysis. Stoke is the most common cause of upper motor neuron syndrome that follows damage to the direct and indirect descending corticospinal fibers. Spasticity develops a few days or weeks after the stroke. What treatment could be used locally in flexor muscles to prevent acetylcholine from being released from its vesicles and reduce hypertonia because of post-stroke spasticity? 6. In this patient (see above), the drug is thought to act within 3-5 days and last 1-3 months. Repeated injections in the distal joints appears safe. Why is use of this drug a concern with regards to use in children with cerebral palsy limb spasticity? A) Can cause muscle weakness, and the drug is no longer functional. B) Can cause pain in arms that can't be reversed easily. C) Concerned that it will cause bronchospasm and exacerbate asthma. D) Can cause bronchitis and impair respiratory health. E) Can migrate out of the specific region it is meant resulting in paralysis of the esophagus or air-way passages. F) Doesn't work for spasticity caused by cerebral palsy.

E

11) A 40-year-old woman, admitted to the emergency hospital after a car collision, had multiple surgeries to repair her injuries and was placed on a ventilator set to assist respiration. On day 13 she was scheduled for a transesophageal ECG, and was given a muscle relaxant IV to facilitate the procedure. Soon afterwards the patient suffered a cardiac arrest and CPR was performed successfully. The potassium recorded during arrest peaked at 7.3 mEq/L. Which of the following muscle relaxant was most likely administered? A) Cisatracurium B) Baclofen C) Tubocurarine D) Dantrolene E) Succinylcholine

E (Katzung, pp 439, Brunton, pp 226, Huggins pp 693) Succinylcholine can release rapidly potassium from intracellular sites. In a normal patient this results in a small, transient hyperkalemia. In patients with extensive soft-tissue trauma or burns, cerebral vascular accident and prolonged stays in an intensive care unit, this hyperkalemia would be much more pronounced.. The cause of succinylcholine-induced hyperkalemia in these patients seems related to up regulation of Nm receptors in the skeletal muscle. Patients are deprived of neural influence or activity because of prolonged immobilization. Such deprivation stimulates the synthesis of new Nm receptors across the muscle membrane. When these receptors are activated by succinylcholine an action potential occurs, ion channels open allowing Na+ to flow inside and K+ to flow outside of the cell. Since succinylcholine is metabolized more slowly than acetylcholine the receptor activation is more prolonged, causing a larger influx of Na+ and a vigorous efflux of K+. When Nm receptors are increased in skeletal muscle this K+ efflux can cause a life-threatening hyperkalemia. A, B, C) Nondepolarizing neuromuscular blockers cause blockade, not activation, of Nm receptors. Therefore they do not induce hyperkalemia. D) Dantrolene has no effect on Nm receptors.

35 A 35-year-old Sami man from Finland has increasing lower back pain for 6 years. On examination he has decreased mobility of his spine. Radiographs show loss of lumbar lordosis, narrowing with sclerosis of sacroiliac joints bilaterally, and syndesmophyte formation at the junction of vertebrae and annulus fibrosus of intervertebral discs. Which of the following complications is most likely to develop in this man? A Glomerulonephritis B Hepatitis C Malabsorption D Meningitis E Uveitis

E Ankylosing spondyloarthritis has a very strong association with the HLA-B27 genotype that has a high prevalence among the native peoples of the circumpolar arctic and subarctic regions of Eurasia and North America. This progressive disease typically involves the lower back and pelvis. The radiographic feature of bamboo spine is characteristic. Spondyloarthropathies can have extra-articular manifestations such as anterior uveitis and aortitis. Glomerulonephritis is most likely a complication of collagen vascular diseases such as systemic lupus erythematosus. Autoimmune hepatitis is not related to joint diseases. Malabsorption is more likely to accompany diarrhea with enteropathic arthritis with infectious etiology. Meningitis may accompany Lyme disease.

48 A 33-year-old man has noticed a lump over his right flank. The lump is painless and has enlarged slowly over the past 3 years. On physical examination, a soft 2-cm nodule is palpable in the subcutis of the right flank above the iliac crest. The lesion is excised. Grossly, it is circumscribed and has a uniformly yellow cut surface. Which of the following is the most likely prognosis for this lesion? A Antibiotic therapy will be needed B Family members will develop similar lesions C Metastases to regional lymph nodes will occur D More skin lesions will occur over time E No recurrence is expected

E Lipoma is the most common benign soft-tissue neoplasm. Such masses are extremely well differentiated and discrete. Multiple lipomas may be seen in some familial cases, but these are rare. These benign tumors do not metastasize; mesenchymal neoplasms do not often metastasize through lymphatics. Recurrence of some atypical lipomas or liposarcomas is possible, but benign lipomas do not recur. Secondary infection of this uncomplicated excision procedure is unlikely. PBD9 1220 BP9 792 PBD8 1249-1250 BP8 832

13 A 38-year-old man has had chronic leg pain for the past 4 months. He passed a urinary tract calculus a month ago. On physical examination, there is local swelling with tenderness just below the right patella. A radiograph of the right lower leg shows a 4-cm cystic area in the right tibial diaphysis without erosion of the cortex or soft-tissue mass. Laboratory studies show serum calcium is 12.6 mg/dL, and phosphorus is 2.1 mg/dL. A biopsy specimen of the lesion is taken and microscopically shows increased osteoclasts and fibroblast proliferation. Which of the following underlying conditions is most likely to account for these findings? A Chronic glomerulonephritis B Chronic osteomyelitis C Giant cell tumor of bone D Paget disease of bone E Parathyroid adenoma

E Parathyroid adenomas secrete parathyroid hormone (PTH) and cause primary hyperparathyroidism with high serum calcium and low phosphate. Excessive secretion of PTH activates osteoclastic resorption of bone. Microfractures within the areas of bone resorption give rise to hemorrhages; this causes an influx of macrophages and, ultimately, reactive fibrosis. These lesions can become cystic, and they are sometimes called brown tumor of bone. Because they contain osteoclasts and fibroblasts, these lesions can be confused with primary bone neoplasms, such as giant cell tumor of bone. Chronic osteomyelitis rarely produces such a discrete lesion. Secondary hyperparathyroidism is seen in patients with chronic renal failure, but the serum phosphorus should be high. Giant cell tumors occur in the epiphysis or metaphysis, however, and they contain plump stromal cells, not fibroblasts. This patient is too young to have Paget disease of bone, which may be osteolytic in its early phase, but is not associated with hypercalcemia.

53 A 6-year-old boy complains of discomfort in the right upper neck that has worsened over the past 6 months. On physical examination, a 5-cm firm mass is palpable in the right lateral neck. The mass is not tender or warm. A biopsy is taken and the microscopic appearance of this mass is shown in the figure. Immunohistochemical staining for which of the following antigens is most likely to be positive in the cells of this lesion? A CD3 B Cytokeratin C Dystrophin D Neuron-specific enolase E Vimentin

E Rhabdomyosarcoma is the most common sarcoma in children. Sarcomas mark with antibody to vimentin, an intermediate cytoplasmic filament, with immunohistochemistry. Note the neoplastic spindle-shaped but markedly pleomorphic cells that have pink cytoplasm with a hint of striations mimicking skeletal muscle cells. CD3 is a T lymphocyte marker. Cytokeratin is a marker for tumors of epithelial origin (e.g., carcinomas). Dystrophin is a membrane-stabilizing protein in striated muscle; it is absent in Duchenne muscular dystrophy. Neuron-specific enolase is a marker of neoplasms with neural differentiation.

54 A 66-year-old woman has experienced pain in the area around the left knee for the past 6 weeks. She can recall no trauma to the leg. On physical examination, no mass is palpable; there is no warmth or swelling, and there is no loss of range of motion. MRI shows a well-circumscribed 4-cm mass superior and inferior to the patella. The mass is within soft tissue, without bony erosion. A biopsy of the mass is obtained and on microscopic examination shows a biphasic pattern of spindle cells and epithelial cells forming glands. Karyotypic analysis of tumor cells shows a t(X;18) translocation. What is the most likely diagnosis? A Desmoid tumor B Leiomyosarcoma C Mesothelioma D Osteoblastoma E Synovial sarcoma 415

E Synovial sarcomas account for 10% of all adult sarcomas and can be found around a joint or in deep soft tissues because they arise from mesenchymal cells, not synovium. Most synovial sarcomas show the t(X;18) translocation. A desmoid tumor is a fibromatosis composed of fibroblasts and collagen. Leiomyosarcomas do not have a biphasic pattern microscopically and are rarely seen in soft tissues. A mesothelioma can be biphasic, but it more typically arises in the pleura, or less commonly the mesothelial surface of peritoneum or pericardium. An osteoblastoma is a bone mass that arises in the epiphyseal region. PBD9 1223-1224 BP9 795-796 PBD8 1254-1255 BP8 835

6 A 17-year-old primigravida gives birth prematurely to an infant small for gestational age. The infant has immediate respiratory distress. Newborn examination shows limb shortening, frontal skull bossing, and small thorax. A radiograph shows normal bone density without fractures. What is the most likely diagnosis? A Achondroplasia B Congenital syphilis C Osteogenesis imperfecta D Rickets E Thanatophoric dysplasia

E Thanatophoric dwarfism is a rare condition resulting from FGFR3 gene mutations. The markedly reduced thoracic size leads to pulmonary hypoplasia, the rate-limiting step to survival. The other listed options can lead to appearance of bone abnormalities at birth, but with the exception of the perinatal lethal form of osteogenesis imperfecta, are not life threatening.

22 A 23-year-old man has had pain in the area of the right knee for the past year. On physical examination, there is point tenderness in a 2-cm focal area just below the patella laterally over the tibia. A radiograph of the right leg shows a 3-cm, broad-based excrescence projecting from the metaphyseal region of the upper tibia. The lesion is excised. The figure shows the gross appearance of the sectioned lesion. What is the most likely diagnosis? A Enchondroma B Fibrous dysplasia C Giant cell tumor D Osteoblastoma E Osteochondroma 409 image

E The figure shows an osteochondroma with glistening cartilaginous cap overlying cancellous bone. This tumorlike condition, also called an exostosis, is benign and, when solitary, is essentially an incidental finding because a sarcoma rarely arises from an osteochondroma. Multiple osteochondromas can be part of an inherited syndrome, however, and onset can be in childhood, accompanied by bone deformity and an increased risk of development of a sarcoma. Both hereditary and sporadic osteochondromas have loss of heterozygosity in EXT1 or EXT2 genes. An osteochondroma is a projection of the cartilaginous growth plate with proliferation of mature bone capped by cartilage. When skeletal growth ceases, osteochondromas tend to cease proliferation as well. They may produce local irritation and pain. An enchondroma is a benign tumor of hyaline cartilage that arises in the medullary space of young adults. Fibrous dysplasia is a localized area of developmental arrest of bone formation. A giant cell tumor is a benign but locally aggressive lesion that arises in the epiphysis of the long bones of young adults and has a "soap bubble" radiographic appearance. An osteoblastoma is a large osteoid osteoma, which can arise in epiphyseal lesions and cause intense pain.

21 A 16-year-old boy has had pain around the right knee for the past 3 months. There are no physical findings except for local pain over the area of the distal right femur. A radiograph of the right leg shows an ill-defined mass involving the metaphyseal area of the distal right femur, and there is elevation of the adjacent periosteum. A bone biopsy specimen is obtained and on microscopic examination shows large, hyperchromatic, pleomorphic spindle cells forming an osteoid matrix. Which of the following tumor suppressor genes is most likely to be mutated in this boy? A APC B BRCA1 C NF1 D PTEN E RB image

E The osteoid production by a sarcoma is diagnostic of osteosarcoma. The metaphyseal location in a long bone, particularly in the region of the knee, is consistent with osteosarcoma, as is the presentation in a young individual. Sporadic cases of osteosarcoma must have loss of both alleles of RB1, whereas in familial cases there is inheritance of one bad copy, and in those cases retinoblastomas are likely to appear first. Mutations of TP53 are found in many cancers, including osteosarcomas, and with the rare familial Li-Fraumeni syndrome. APC mutations are seen in association with colonic adenocarcinomas. BRCA1 mutations are associated with breast and ovarian cancers. NF1 mutations lead to neurofibromatosis type 1, with many neural and soft tissue tumor types, but less likely primary bone tumors. PTEN mutations are most often associated with endometrial and prostate cancers.

8 A 77-year-old woman trips on the carpet in her home and falls to the floor. She immediately has marked pain in the right hip. On physical examination, there is shortening of the right leg with external rotation and marked pain with any movement. A radiograph shows a right femoral neck fracture. The fracture is repaired. Six months later, a dual-energy x-ray absorptiometry (DEXA) scan of the patient's left hip and lumbar vertebrae shows bone mineral density 2 standard deviations below the young adult reference range. Which of the following cellular processes contributes most to development of her findings? A Decreased secretion of interleukin-6 by monocytes B Increased sensitivity of osteocytes to parathyroid hormone C Insensitivity of bone matrix to 1,25-dihydroxycholecalciferol D Mutation in the fibroblast growth factor receptor 3 gene E Increased osteoclast activity F Synthesis of chemically abnormal osteoid matrix

E With advancing age, the ability of osteoblasts to divide and to lay down osteoid is reduced, whereas in some persons osteoclast activity increases, giving rise to accelerated bone loss known as osteoporosis. Differentiation 416of stromal progenitor cells into osteoclasts requires binding of RANK ligand on osteoblasts to RANK receptor on osteoclast precursors and stimulation by M-CSF produced by osteoblasts. Osteoprotegerin (OPG) is a "decoy receptor" for RANK ligand that slows osteoclast formation and action. When osteoblasts produce less OPG, bone loss is accelerated. Marrow stromal cells produce WNT proteins that bind to osteoblast receptors, activating β-catenin and OPG production. Postmenopausal osteoporosis is characterized by hormone-dependent acceleration of bone loss. Estrogen deficiency results in increased secretion of interleukin-1, interleukin-6, and tumor necrosis factor-α by monocytes-macrophages. These cytokines act by increasing the levels of RANK and RANKL, and decreasing the levels of OPG. In older women, bone loss is accelerated by reduced synthesis and increased resorption. Nonhormonal drugs such as bisphosphonates are designed to reduce osteoclast resorption of bone.

18) A 37-year-old woman, who has been suffering from multiple sclerosis for two years, complained of increased painful cramp in her legs. The neurologist prescribed baclofen, 3 tablets daily. Which of the following adverse effects would be most likely to occur in this patient? A) Hallucinations B) Ataxia C) Dystonic reactions D) Hypotension E) Drowsiness

E. Answer: E (Katzung, pp 441, Brunton, pp 543) Baclofen is a GABAB receptor agonist that is currently used to treat various forms of spasticity. The drug causes hyperpolarization of neuron membrane, which in turn reduced the release of excitatory neurotransmitters in the brain as well as in the spinal cord. The most common adverse effects of baclofen are relate to this central inhibitory activity. Drowsiness is by far the most common, affecting about one half of patients. A, B, C, D) All these listed effects can occur with baclofen but they are very rare at standard therapeutic doses.

a primitive neuroectodermal tumor (PNET) of children under 20 yrs. Diaphysis, invades cortex & raises the periosteum --> onion-skin layers of new bone around the diaphysis (# x-ray sign) sheets of small round cells (may resemble lymphoma or neuroblastoma)& occasional Homer-Wright pseudorosettes may be mistaken for acute osteomyelitis combined therapy --> 75% 5-year survival

EWING'S SARCOMA

no longer a first‐line approach for the treatment of osteoporosis in postmenopausal women due to the increased risk of stroke, thromboembolism, and coronary artery disease.

Estrogen replacement

• Uses: Moderate to severe Rheumatoid arthritis; psoriatic arthritis; juvenile arthritis • Mechanism of action:-It inhibits the binding of TNF-alpha and TNF-beta to their receptors, thus decreasing the inflammatory actions of TNF-alpha • Adverse effects: o Injection site reaction(erythema, pruritis, pain and swelling) o Autoantibody formation o Hypersensitivity reactions; o Serious infections; o Increased risk of cancer o Contraindications: documented hypersensitivity; sepsis; concurrent live vaccination. Patients with MS should avoid this drug

Etanercept

It consists of the extracellular domain of the TNF-alpha receptor coupled to human IgG1

Etanercept

• It binds to circulating TNF-alpha making it biologically inactive and preventing it from interacting with cell surface TNF receptors that would lead to cell activation. • It is given by subcutaneous injection once weekly by the patient. It has a long half-life in comparison with Anakinra. It binds to both the soluble and membrane-bound form of TNF- alpha

Etanercept

19 A 12-year-old girl has had sudden onset of severe pain in her left knee that has awakened her from sleep on several occasions during the past 6 weeks. For each episode, her mother has given her acetylsalicylic acid (aspirin), and the pain has been relieved. On physical examination, there are no remarkable findings. A radiograph of the left knee shows a well-defined, 1-cm lucent area surrounded by a thin rim of bony sclerosis located in the proximal tibial cortex. The patient undergoes radioablation of the lesion, and the pain does not recur. What is the most likely diagnosis of this lesion? A Enchondroma B Fibrous dysplasia C Giant cell tumor D Osteoblastoma E Osteochondroma F Osteoid osteoma image

F An osteoid osteoma is a benign tumor of the bone with a central nidus of woven bone and sclerotic rim. It most often occurs in children and young adults. Pain disproportionate to the size of the tumor is characteristic. If such a lesion is larger than 2 cm, it is classified as an osteoblastoma, and most of these arise in vertebral posterior elements. It can be treated effectively by curettage. The acute pain is mediated by release of prostaglandins, so aspirin is an effective analgesic. An enchondroma is a benign tumor of hyaline cartilage that arises in the medullary space in young adults. Fibrous dysplasia is a localized area of developmental arrest of bone formation. A giant cell tumor is a benign but locally aggressive lesion that arises in the epiphysis of the long bones of young adults and has a "soap bubble" radiographic appearance. An osteochondroma is a projection of the cartilaginous growth plate to form an exostosis.

Two overlapping lesions, occur in old age, in soft tissues and in bones, as grey-white infiltrative tumors. Fibrosarcoma; malignant spindle cells in "herring-bone" pattern; while MFH; maligt. spindle cells in "storiform" pattern + giant cells & phagocytic cells (thought to be histiocytic, but actually fibroblasts). Prognosis depends upon cytologic grade; (20% 5- year survival for high grade MFH) & and upon clinical Stage (size of tumor & extent of spread).

FIBROSARCOMA / MALIGNANT FIBROUS HISTIOCYTOMA (MFH)

• Non-purine selective inhibitor of xanthine oxidase • It works by non-competitively blocking the molybdenum pterin center which is the active site on xanthine oxidase. • more potent & less effective than allopurinol • Side Effects: Nausea, diarrhea, arthralgia, headache, increased hepatic serum enzyme levels and rash • Interactions: Same as for allopurinol

Febuxostat

? hamartomatous, sharply defined, radiolucent, fibrous lesion, in the metaphyseal cortex of long bones of children; painless; single or multiple; extremely common & disappear spontaneously.

Fibrous Cortical Defect (Non-ossifying Fibroma):

localized replacement of bone by fibro-osseous proliferations (trabeculae of woven bone not lined by osteoblasts & forming "Chinese letters". May be: 1. Monostotic (70%), 2. polyostotic (25%), and 3. polyostotic disease + endocrinopathies (McCune- Albright syndrome = skin pigmentation - large café-au-lait macules-& precocious puberty). Rarely, malignant transf.

Fibrous Dysplasia

Nodular fasciitis -- limbs, chest, back-- rapidly growing --very cellular with mitoses - looks malignant! Myositis ossificans- fibroblastic proliferation plus metaplastic (benign) bone; cured by simple excision fibromatosis; - Plantar or palmar, desmoid eg. abdominal ( pregnancy) Fibroma and fibrosarcoma; Malignant fibrous histiocytoma

Fibrous tumors and tumor- like lesions

hyperparathyroidism, primary : (autonomous hyperplasia or tumor(80%,- adenoma)of parathyroid) / secondary :(compensatory hypersecretion of PTH due to prolonged hypocalcemia) increased osteoclastic activity --> bone resorption, and peritrabecular fibrosis marrow fibrosis & reparative giant cell granulomas (brown tumors) --> cysts within the bone (osteitis fibrosa cystica) loosening of teeth, and rarefied bones of distal phalanges & clavicles on x-ray.

Generalized Osteitis Fibrosa Cystica

argoilism)- severe, death by 12 years;

Hurlers

Serious adverse effects: • Hypersensitivity reactions, • Serious infections such as TB may develop. • There is a risk of hepatosplenic lymphoma, a T- cell lymphoma. • All patients who developed this cancer were on Azathioprine as well. • Heart failure.

Infliximab

• Human/mouse chimeric IgG1 monoclonal antibody, possessing human constant (Fc) regions and murine variable regions, against TNF-alpha • binds both bound and soluble TNF-alpha • has a higher affinity for TNF-alpha than other biological drugs • It is given by IV infusion in combination with oral methotrexate at 0, 2, and 6 weeks, and after that once every 8 weeks • Infusion reactions including fever, chills, pruritis and rash may occur within 1 or 2 h after giving the infusion • acetaminophen, diphenhydramine or corticosteroids may be given

Infliximab

• Uses: Crohn's disease, Ulcerative colitis, Rheumatoid Arthritis, Ankylosing spondylitis, Psoriatic arthritis, IBD. When used in the treatment of RA, it is used together with methotrexate unless contraindicated , or not tolerated by the patient • Mechanism of action: It binds to soluble and transmembrane forms of TNF-alpha and prevents it from interacting with its receptors on various cells • Adverse effects: common-abdominal pain, fatigue, headache, infusion reaction. Upper respiratory and urinary tract infections may occur

Infliximab

Uses: moderate to severe RA and psoriatic arthritis Mechanism of action: inhibits pyrimidine synthesis as it inhibits dihydro-orotate dehydrogenase the rate -limiting step for de novo pyrimidine synthesis It therefore inhibits T and B lymphocytes whose proliferation depends on the de novo pathway. It is given as loading dose of 100mg daily for 3 days followed by a daily maintenance dose of 20mg. • The drug requires to be metabolized to active metabolites.

Leflunomide

Most common toxicities associated with long-term methotrexate therapy:- • GI disturbances nausea, vomiting, diarrhea, anorexia • Leucopenia • MTX lung, a hypersensitivity reaction that can develop any time during therapy-but mostly within a year of starting MTX. • Hepatotoxicity which correlates with accumulative dosing. Note that low dose methotrexate given for long-term treatments(years) increases the risk of hepatic fibrosis • Tests: CBC's and LFT's (Liver Function Tests) are mandatory .

Methotrexate

• Reduces the signs and symptoms of RA and slows the rate of radiographic changes • It is the most commonly prescribed DMARD. It may be used as a monotherapy, but is usually the anchor drug in combinations with other drugs such as anti-TNF-alpha agents • It is given at much lower doses than those used in anti- cancer regimens. The route may be oral, given once a week or once every other week. It is given in conjunction with folate as adjuvant therapy.

Methotrexate

shortest-duration non-depolarizing neuromuscular-blocking drug[1] or skeletal muscle relaxant in the category of non-depolarizing neuromuscular-blocking drugs,[2] used adjunctively in anesthesia to facilitate endotracheal intubation[3] and to provide skeletal muscle relaxation during surgery or mechanical ventilation.

Mivacurium

Lysosomal storage disease; defect in x degradation due to abn. deficiency of enzymes and resulting accumulation of dermatan sulfate, heparan sulfate, keratan sulfate and chondroitin sulfate--- result - severe somatic and neurological changes

Mucopolysaccharidosis

giant osteoid osteoma, in cancellous bones (eg. vertebrae), but no sclerotic rim, painless & tends to recur

Osteoblastoma

synthesize and transport protein (like type 1 collagen) and osteoprotegerin, initiate mineralization; have receptors for parathyroid hormone, Vit.D and estrogen, cytokines; produce many growth factors ( like Interleukin- 1), and RANK-L

Osteoblasts

(Exostosis): Benign cartilage - capped outgrowth attached to the underlying skeleton by a bony stalk. May be AD 'multiple hereditary exostosis'; <1% -sarcoma.

Osteochondroma

maturation req IL, TNF. Contain RANK receptors.

Osteoclasts

regulate daily serum Ca++ and P

Osteocytes

Deficiency, or abnormality in synthesis,of type I collagen (90-95% of bone matrix) Result:-"Osteopenia"; thining of both cortex & trabeculae -- "too little bone" - Increased predisposition to fractures effects other anatomic structures rich in type 1 collagen such as joints, eyes, ears. skin and teeth. Of the four major subtypes : type II disease is fatal early , type 1 is compatible with long survival

Osteogenesis Imperfecta

Histology:Interconnecting trabeculae of woven bone rimmed by osteoblasts. NIDUS is sur.by dense reactive bone.

Osteoid Osteoma

adolescent males, painful, benign, small (<1cm), cortical, radiolucent focus with sclerotic rim.Pain caused by xs prostaglandin E2 --relieved by aspirin

Osteoid Osteoma Histology:Interconnecting trabeculae of woven bone rimmed by osteoblasts. NIDUS is sur.by dense reactive bone.

benign, densely sclerotic, in the cortical bones of skull & face; composed of woven and lamellar bone;may be associated with Gardner's syndrome

Osteoma

Generalized pain Fractures - including compression fractures Mottled areas of lucency (pseudofractures) in bones -eg pelvic bones -Loosers zones, Milkmans syndrome Look for reason for decerase levels of vitamin D -: no sun; malabsorption; kidney disease; fad diets, etc

Osteomalacia

infarction of bone, due to: fractures (vascular interruptions), vessel injury (vasculitis, radiation ), thrombosis and embolism -dysbarism, Sickle cell anemia; & steroid- induced Medullary infarcts affect marrow trabeculae & fat, but spare the cortex (collaterals), Subchondral ( wedge- shaped)--> collapse of trabecular and articular cartilage --(typically head of femur) effects: Bone pain, joint pains & second.osteoarthritis

Osteonecrosis

defined as a T‐score of ‐2.5 or less

Osteoporosis

teriparatide

PTH analog

cause osteoblast precursors to express the osteoclast differentiation factor RANK‐ligand (RANKL).

PTH, shear stress, and transforming growth factor beta (TGF‐beta)

Etiology : Possibly caused by slow virus infection effecting osteoblasts & osteoclasts (paramyxovirus), More likely : GENETIC : More common among first degree relatives and up to 40% show autosomal dominant pattern Most common in Males, 5th decade; common in England, USA, Western Europe; rare in China, Japan and Africa three stages: 1. an initial osteolytic stage, 2. a mixed osteolytic-osteoblastic stage & 3. a burnt-out quiescent osteosclerotic stage. monostotic (15%), polyostotic (85%) histo: large osteoclasts; "jigsaw" (mosaic) pattern of bone; later prominant osteoblasts-osteosclerosis & increased vascularity, Coarse but weak bone

Paget disease

Increased Alk. Phosph. and urinary Hydroxyproline. effects: bone deformities, fractures, cranial nerve compression , osteoarthritis, A-V shunt effect -- high output heart failure. Tumors- Benign-Giant cell tumor; malignant- osteosarcoma, chondrosarcoma.-long bones,pelvis,skull Treatment: calcitonin, Biphosphanates

Paget disease

• Cutaneous lesions • **Blood dyscrasias-bone marrow suppression which may be fatal** • -Autoimmune syndrome • -Taste disturbance-bitter taste-loss of sugar/salt taste • -Proteinuria (nephrosis-degeneration of renal tubular epithelium) • -Penicillamine may be immunogenic and produce hypersensitivity reactions. -Patients with Goodpastures Syndrome, if given penicillamine, may die.

Penicillamine

• Usually given if other drugs have failed in patients with active RA. It is an antiarthritic whose major use is the treatment of Wilson's disease as it chelates copper. • Mechanism of action - uncertain • Pharmacokinetics This drug is taken orally, plasma peaks 1-2 hours later, but the clinical effectiveness takes months. • Before therapy and every 2-4 weeks during therapy, measurement of platelets, urinalysis, and CBC are performed

Penicillamine

blood-borne infections; 85%-staph. aureus Pseud. and Klebsiella --drug abusers; H. Influenzae and grp. b strep.---children; Salmonella----sicklers. Fever, throb. pain; x-ray-lytic lesion pH and O2 tension decreased by bact. infection ischemic necrosis of bone --> sequestrum subperiosteal new bone on outside of affected bone --> involucrum treatment: antibiotics + surgery; 10%--chronic complications: septicemia, endocarditis,fracture, amyloidosis & sq. c. ca. in sinus tracts.

Pyogenic Osteomyelitis

receptor activator of nuclear factor-kappa

RANK

secreted by osteoblasts in response to a variety of hormonal and cytokine signals known to be important to bone metabolism, and is the principal regulator of the differentiation and activity of osteoclasts

RANKL

Pathophysiology: renal failure --> phosphate retention - -> hypocalcemia --> Assoc. 2ndy. hyperparathyroidism -- increased osteoclastic activity ---(osteitis fib. cystica) renal failure --> 1,25-(OH)2-D3 deficiency-> hypocalcemia

Renal Osteodystrophy

thought to be the result of hyperparathyroidism secondary to hyperphosphatemia combined with hypocalcaemia, both of which are due to decreased excretion of phosphate by the damaged kidney. Low activated vitamin D3 levels are a result of the damaged kidneys' inability to convert vitamin D3 into its active form, calcitriol, and result in further hypocalcaemia.

Renal Osteodystrophy

Chronic systemic inflammatory disease, immunologically mediated, effecting numerous organs including joints Typically: woman, 3rd, 4th decade chronic synovitis --> joint ankylosis & deformities mostly affects: small joints of hands & feet, then wrists, elbows, ankles & knees, symmetrically. Rheumatoid nodules. Serum rheumatoid factor; X-ray: osteopenia, narrowed joint space , erosions; typical deformities Synovial proliferation (pannus), intense lympho- plasmacytic infiltrate, release of destructive enzymes & cytokines eg.-TNF's--> destruction of articular cartilage & fibrous then bony ankylosis. pathogenesis: autoimmune reaction in genetically susceptible individual (disease common in monozygotic twins); but initial acute disease may be the result of microbial antigen ( ?EBV ?retrovirus ?mycoplasma ). Continuing autoimmune reaction; 80% are HLA-DR4, or DR1. activates CD4+ T cells and also B cells --- > autoantibodies against autologous IgG (RF = usually IgM, but may be IgG, IgA or IgE).

Rheumatoid Arthritis (RA)

Rhabdomyoma -rare eg. cardiac Rhabdomyosarcoma- common in childhood--aggressive; "cross striations"

Skeletal muscle tumors

Distribution: 40% in the lower extremeties; 30% in the trunk & retroperitoneum; 20% in the upper extremeties & 10% in the head & neck. Prognosis depends upon: 1. Site, 2. Size, 3. Clinical Stage , 4. histologic type & 5. Grade.

Soft Tissue Tumors (STT)

Sulfonamide plus salicylate

Sulfasalazine

Adverse effects: • GI adverse effects such as nausea and vomiting, diarrhea and anorexia are the most common. • Rash and serum sickness -like reactions can be managed with antihistamines and if necessary with corticosteroids in the short run. If a hypersensitivity reaction occurs, therapy should be stopped, and another DMARD substituted • Leukopenia, thrombocytopenia, alopecia, stomatitis and elevated hepatic enzymes may occur. These hematological abnormalities usually occur within the first 3-6 months of treatment, and then abate. They are reversible when treatment is stopped.

Sulfasalazine it is a combination of sulfapyridine and 5-aminosalicylic acid, • Its activity is due to the active metabolite-5-aminosalicylic acid which is also called mesalamine, and sulfapyridine.

• Uses It is a prodrug used in the treatment of ulcerative colitis, IBD, and rheumatoid arthritis. Iand it is metabolized to its constituents by bacteria in the gut. • Mechanism of action The anti-inflammatory actions of mesalamine are thought to be due to the inhibition of cyclooxygenases in the gut with the diminished production of prostaglandins thus reducing colonic inflammation.

Sulfasalazine it is a combination of sulfapyridine and 5-aminosalicylic acid, • Its activity is due to the active metabolite-5-aminosalicylic acid which is also called mesalamine, and sulfapyridine.

causes osteoclast precursors to differentate into mature osteoclasts.

The RANKL‐RANK binding interaction, together with macrophage colony stimulating factor (M‐CSF)

14) A 35-year-old male was hospitalized with the diagnosis of schizophrenia and was started on a high dose of a neuroleptic. After 2 days of therapy he became rigid, appeared confused at times and had a fever of 104E F. The physical examination reveals tachycardia, labile blood pressure, profuse diaphoresis, sialorrhea, dyspnea and urinary incontinence. Which of the following drugs should be immediately given IV to this patient? A) Atropine B) Succinylcholine C) Dantrolene D) Dopamine E) Prazosin

The clinical picture of the patient strongly suggest the diagnosis of neuroleptic malignant syndrome, a rare disorder very similar to the malignant hyperthermia induced by some neuromuscular blocking agents (mainly succinylcholine) and inhalational anesthetics. In malignant hyperthermia the uncontrolled release of CA++ from the sarcoplasmic reticulum of skeletal muscle is the initiating event. Dantrolene, a drug that blocks the release of calcium from sarcoplasmic reticulum is the drug of choice in malignant hyperthermia. The drug is also currently given in case of neuroleptic malignant syndrome, even if the association of the syndrome with a defect of Ca++ metabolism in skeletal muscle is less clear. A , D, E) These drugs are dangerous in this patient since they would worsen the tachycardia. B) Succinylcholine administration would be completely irrational since the drug can cause malignant hyperthermia.

27 A 14-year-old girl experiences severe pain in the right leg after performing a gymnastic floor exercise. On physical examination, there is marked pain on palpation of the right lower thigh just above the knee. Radiographs show a pathologic fracture across a 3-cm lower femoral diaphyseal lesion that has central lucency with a thin sclerotic rim. The lesion is completely intramedullary and well circumscribed. A bone biopsy specimen of the affected region is taken and microscopically shows scattered trabeculae of woven bone in a background of fibroblastic proliferation. What is the most likely diagnosis? A Ewing sarcoma B Fibrous dysplasia C Fracture callus D Osteogenic sarcoma E Osteoid osteoma

This single focus (monostotic) fibrous dysplasia weakens the bone to the point of pathologic fracture. This benign tumorlike condition is uncommon. The histologic appearance of woven bone in the middle of benign-looking fibroblasts is characteristic. Seventy percent of cases are monostotic, and the ribs, femur, tibia, mandible, and calvaria are the most frequent sites of involvement. Local deformity and, occasionally, fracture can occur. Polyostotic fibrous dysplasia may involve craniofacial, pelvic, and shoulder girdle regions, leading to severe deformity and risk for fracture. Ewing sarcoma usually occurs in the diaphyseal region of the long bones and is identified histologically by sheets of small, round cells. A fracture callus should not be so localized within bone, and should not develop so quickly following trauma. An osteosarcoma is typically a large destructive lesion without central lucency. An osteoid osteoma has a small central nidus with surrounding sclerosis.

T/F: Allopurinol decreases the metabolism of azathioprine and mercaptopurine. Doses of azathioprine and mercaptopurine must be reduced if given together with allopurinol as they are highly toxic .

True

T/F: Normally, FGF-mediated activation of FGFR3 inhibits cartilage proliferation; in achondroplasia, the mutations cause constitutive activation of FGFR3 and thereby suppress growth.

True

Blood borne-from lung or elsewhere;AIDS; Destructive and difficult to control; Pain, low grade fever, weight loss example: Pott's disease (TB spondylitis) flare-up of post-primary dissemination foci in vertebral bodies, sparing the lamina & spines (necrosis extends to the discs) Most commonly affects thoracic and lumbar vertebrae caseating granulomas with no reactive bone formation --> collapse of vertebrae & 1. angular kyphoscoliosis ( hunchback), 2. cold abscess (psoas) or presentation as inguinal "hernia" & 3. paraplegia

Tuberculous Osteomyelitis

a naturally occurring mono-quaternary alkaloid obtained from the bark of the South American plant Chondrodendron tomentosum, a climbing vine known to the European world since the Spanish conquest of South America

Tubocurarine

Benzylioquinolines

Tubocurarine, Mivacurium. Cisatracurium

Can have stereoisomers in this group. First generation has histamine release.

Tubocurarine, Mivacurium. Cisatracurium Benzylioquinolines

Most common form of skeletal malignancy May be from direct extension, via lymphatics or vascular, or intraspinal seeding Vasc.---breast, prostate, lung, kidney,thyroid may be lytic (kidney, lung) or blastic(prostate) or commonly mixed. Lytic lesion caused by tumor cells secreting protaglandins, parathyroid- related protein etc. Blastic mets stimulate osteoblasts.

Tumors Metastatic to the Skeleton

leiomyoma -benign--common leiomyosarcoma--malignant--rel. uncommon--- -cigar-shaped nuclei; interweaving fascicles

Tumors of smooth muscle

a muscle relaxant in the category of non-depolarizing blocking agents. indicated as an adjunct to general anesthesia, to facilitate endotracheal intubation and to provide skeletal muscle relaxation during surgery or mechanical ventilation. Although often thought of as a muscle relaxant, it may be more accurate to classify it as a paralyzing agent.

Vecuronium

little histaminergic, vagolytic, or ganglionic effects

Vecuronium

Estrogen replacement during postmenopausal period is primary prescribed

address vasomotor symptoms.

central acting skeletal muscle relaxants

baclofen, tizanidine, gabapentin, diazepam

block a step in the mevalonate pathway. Disruption of this process decreases prenylation, the covalent attachment of certain lipids (farnesyl) to multiple proteins, including intracellular regulatory proteins such as GTPases. This impairs a number of osteoclastic functions including H+‐ATPase activity, and ultimately causes osteoclast apoptosis.

bisphosphonates

can be given intranasally or subcutaneously for osteoporosis

calcitonin patients stop responding after 1 week

muscle acting skeletal muscle relaxants

dantrolene, botox

benign; encapsulated aggregated adipocytes; variants- angiolipoma,spindle cell lipoma

lipoma

malignant; recognized by characteristic histology and presence of lipoblasts. Main types- Myxoid (commonest), Well differentiated and Pleomorphic

liposarcoma

co-secreted by osteoblasts and serves to modulate the effective levels of RANKL by acting as a decoy on the RANK receptor in the osteoclast.. As expected, animal models confirm that RANKL excess shifts that balance of bone metabolism in the direction of catabolism and causes osteopenia, and conversely that absence of RANKL increases bone density. RANKL inhibition offers the therapeutic possibility to treat osteoporosis.

osteoprotegerin

a typical non-depolarizing curare-mimetic muscle relaxant. It acts as a competitive acetylcholine antagonist on neuromuscular junctions, displacing ACh from its post-synaptic nicotinic receptors

pancuronium onset 60-120 seconds, duration 80-60 mins

may provide protection against cancer, unlike tamoxifen

raloxifene adverse effects = thromboembolism, hot flashes

selective estrogen receptor modulator

raloxifene adverse effects = thromboembolism, hot flashes

severe symptomatic hypocalcemia

slow infusion of calcium gluconate

ostochondral "rachitic rosary"

softening of bones-- fractures and characteristic deformities rickets, scurvy

Steroid like structure but it has no hormone effects.

succinyl choline

depolarizing blocker

succinylcholine

may cause osteosarcoma

teraparitide

reserved for advanced osteoporosis (-3.5)

teraparitide

drugs causing histamine release

tubocurarine

acts as an antagonist for the nicotinic acetylcholine receptor (nAchr), meaning it blocks the receptor site from Ach.[15] This may be due to the quaternary amino structural motif found on both molecules.

tubocurarine can also block Nn receptors and therefore can act as a ganglionic blocking drug, mainly when given in larger doses.

• Second line drugs used in the treatment of RA USES: • RA and SLE treatment • Porphyria cutanea tarda (PCT) familial and nonfamilial - uroporphyrinogen decarboxylase deficiency

• Both of these drugs are 4-aminoquinolines and are antimalarials-used both prophylactically, and in treatment of patients who have contracted malaria Hydroxychloroquine/chloroquine

• The drug suppresses the responsiveness of T- lymphocytes to mitogens • Decreases leukocyte chemotaxis • Stabilizes lysosomal membranes • Inhibits DNA and RNA synthesis • It traps free radicals thus reducing oxidative reactions which may be associated with the deteriorating bone

• Both of these drugs are 4-aminoquinolines and are antimalarials-used both prophylactically, and in treatment of patients who have contracted malaria Hydroxychloroquine/chloroquine

• Slow to act (Months) • No direct analgesic actions • Slow the course of the disease and may prevent further deterioration of joints • Given to NSAID non-responders; non-compliant patients, and those who have serious GIT problems with NSAIDS and Steroids • Given to patients whose joints are deteriorating rapidly • Juvenile RA

• D-penicillamine • Chloroquine and hydroxychloroquine • Sulfonamide plus salicylate=Sulfasalazine


Ensembles d'études connexes

NEURO - CHAPTER 4 NEURAL CONDUCTION AND SYNAPTIC TRANSMISSION: HOW NEURONS SEND AND RECEIVE SIGNALS

View Set

Learning System RN 3.0 Fundamentals Practice

View Set

Biology: life on earth chapter 27, 28, 31, 38, 39

View Set

MGT 3200 - final practice questions

View Set

2. Reologické modely biomateriálů

View Set

Vocabulary Workshop Level B Unit 13 Choosing the Right Word

View Set